Как найти касательную перпендикулярную функции

Уравнение касательной к графику функции

П. Романов, Т. Романова,
г. Магнитогорск,
Челябинская обл.

Уравнение
касательной к графику функции

Статья опубликована при поддержке Гостиничного комплекса «ИТАКА+». Останавливаясь в городе судостроителей Северодвинске, вы не столкнетесь с проблемой поиска временного жилья. Тут, на сайте гостиничного комплекса «ИТАКА+» http://itakaplus.ru, вы сможете легко и быстро снять квартиру в городе, на любой срок, с посуточной оплатой.

На современном этапе развития
образования в качестве одной из основных его
задач выступает формирование творчески мыслящей
личности. Способность же к творчеству у учащихся
может быть развита лишь при условии
систематического привлечения их к основам
исследовательской деятельности. Фундаментом для
применения учащимися своих творческих сил,
способностей и дарований являются
сформированные полноценные знания и умения. В
связи с этим проблема формирования системы
базовых знаний и умений по каждой теме школьного
курса математики имеет немаловажное значение.
При этом полноценные умения должны являться
дидактической целью не отдельных задач, а
тщательно продуманной их системы. В самом
широком смысле под системой понимается
совокупность взаимосвязанных взаимодействующих
элементов, обладающая целостностью и устойчивой
структурой.

Рассмотрим методику обучения
учащихся составлению уравнения касательной к
графику функции. По существу, все задачи на
отыскание уравнения касательной сводятся к
необходимости отбора из множества (пучка,
семейства) прямых тех из них, которые
удовлетворяют определенному требованию
– являются касательными к графику некоторой
функции. При этом множество прямых, из которого
осуществляется отбор, может быть задано двумя
способами:

а) точкой, лежащей на
плоскости xOy (центральный пучок прямых);
б) угловым коэффициентом (параллельный пучок
прямых).

В связи с этим при изучении
темы «Касательная к графику функции» с целью
вычленения элементов системы нами были выделены
два типа задач:

1) задачи на касательную,
заданную точкой, через которую она проходит;
2) задачи на касательную, заданную ее угловым
коэффициентом.

Обучение решению задач на
касательную осуществлялось при помощи
алгоритма, предложенного А.Г. Мордковичем [2].
Его принципиальное отличие от уже известных
заключается в том, что абсцисса точки касания
обозначается буквой a (вместо x0), в связи с чем
уравнение касательной приобретает вид

y = f(a) + f ‘(a)(x – a)

(сравните с y = f(x0) + f ‘(x0)(x
– x0)). Этот методический прием, на наш
взгляд, позволяет учащимся быстрее и легче
осознать, где в общем уравнении касательной
записаны координаты текущей точки, а где
– точки касания.

Алгоритм
составления уравнения касательной к графику
функции y = f(x)

1. Обозначить буквой a
абсциссу точки касания.
2. Найти f(a).
3. Найти f ‘(x) и f ‘(a).
4. Подставить найденные числа a, f(a), f ‘(a) в
общее уравнение касательной y = f(a) = f ‘(a)(x – a).

Этот алгоритм может быть
составлен на основе самостоятельного выделения
учащимися операций и последовательности их
выполнения.

Практика показала, что
последовательное решение каждой из ключевых
задач при помощи алгоритма позволяет
формировать умения написания уравнения
касательной к графику функции поэтапно, а шаги
алгоритма служат опорными пунктами действий.
Данный подход соответствует теории поэтапного
формирования умственных действий, разработанной
П.Я. Гальпериным и Н.Ф. Талызиной [3].

В первом типе задач были
выделены две ключевые задачи:

  • касательная проходит через
    точку, лежащую на кривой (задача 1);
  • касательная проходит через
    точку, не лежащую на кривой (задача 2).

Задача 1. Составьте уравнение
касательной к графику функции в точке M(3; – 2).

Решение. Точка M(3; – 2)
является точкой касания, так как

1. a = 3 – абсцисса точки
касания.
2. f(3) = – 2.
3. f ‘(x) = x2 – 4, f ‘(3) = 5.
y = – 2 + 5(x – 3), y = 5x – 17 – уравнение
касательной.

Задача 2. Напишите уравнения
всех касательных к графику функции y = – x2
– 4x + 2, проходящих через точку M(– 3; 6).

Решение. Точка M(– 3; 6) не
является точкой касания, так как f(– 3)
­ 6 (рис. 2).

1. a – абсцисса точки
касания.
2. f(a) = – a2 – 4a + 2.
3. f ‘(x) = – 2x – 4, f ‘(a) = – 2a – 4.
4. y = – a2 – 4a + 2 – 2(a + 2)(x – a)
– уравнение касательной.

Касательная проходит через
точку M(– 3; 6), следовательно, ее координаты
удовлетворяют уравнению касательной.

6 = – a2 – 4a + 2 – 2(a +
2)(– 3 – a),
a2 + 6a + 8 = 0
^ a1 = – 4, a2 = – 2.

Если a = – 4, то уравнение
касательной имеет вид y = 4x + 18.

Если a = – 2, то уравнение
касательной имеет вид y = 6.

Во втором типе ключевыми
задачами будут следующие:

  • касательная параллельна
    некоторой прямой (задача 3);
  • касательная проходит под
    некоторым углом к данной прямой (задача 4).

Задача 3. Напишите уравнения
всех касательных к графику функции y = x3 – 3x2
+ 3, параллельных прямой y = 9x + 1.

Решение.

1. a – абсцисса точки
касания.
2. f(a) = a3 – 3a2 + 3.
3. f ‘(x) = 3x2 – 6x, f ‘(a) = 3a2 – 6a.

Но, с другой стороны, f ‘(a) = 9
(условие параллельности). Значит, надо решить
уравнение 3a2 – 6a = 9. Его корни a = – 1, a = 3
(рис. 3).

4. 1) a = – 1;
2) f(– 1) = – 1;
3) f ‘(– 1) = 9;
4) y = – 1 + 9(x + 1);

y = 9x + 8 – уравнение
касательной;

1) a = 3;
2) f(3) = 3;
3) f ‘(3) = 9;
4) y = 3 + 9(x – 3);

y = 9x – 24 – уравнение
касательной.

Задача 4. Напишите уравнение
касательной к графику функции y = 0,5x2 – 3x + 1,
проходящей под углом 45° к прямой y = 0 (рис. 4).

Решение. Из условия f ‘(a) =
tg 45° найдем a:  a – 3 = 1
^ a = 4.

1. a = 4 – абсцисса точки
касания.
2. f(4) = 8 – 12 + 1 = – 3.
3. f ‘(4) = 4 – 3 = 1.
4. y = – 3 + 1(x – 4).

y = x – 7 – уравнение
касательной.

Несложно показать, что
решение любой другой задачи сводится к решению
одной или нескольких ключевых задач. Рассмотрим
в качестве примера следующие две задачи.

1. Напишите уравнения
касательных к параболе y = 2x2 – 5x – 2, если
касательные пересекаются под прямым углом и одна
из них касается параболы в точке с абсциссой 3
(рис. 5).

Решение. Поскольку дана
абсцисса точки касания, то первая часть решения
сводится к ключевой задаче 1.

1. a = 3 – абсцисса точки
касания одной из сторон прямого угла.
2. f(3) = 1.
3. f ‘(x) = 4x – 5, f ‘(3) = 7.
4. y = 1 + 7(x – 3), y = 7x – 20 – уравнение первой
касательной.

Пусть a – угол наклона первой
касательной. Так как касательные
перпендикулярны, то – угол наклона второй касательной. Из
уравнения y = 7x – 20 первой касательной имеем tg 
a = 7. Найдем

Это значит, что угловой
коэффициент второй касательной равен .

Дальнейшее решение сводится к
ключевой задаче 3.

Пусть B(c; f(c)) есть точка
касания второй прямой, тогда

1.  – абсцисса второй точки касания.
2. 
3. 
4. 
– уравнение
второй касательной.

Примечание. Угловой
коэффициент касательной может быть найден проще,
если учащимся известно соотношение
коэффициентов перпендикулярных прямых k1•k2
= – 1.

2. Напишите уравнения всех
общих касательных к графикам функций

Решение. Задача сводится к
отысканию абсцисс точек касания общих
касательных, то есть к решению ключевой задачи 1 в
общем виде, составлению системы уравнений и
последующему ее решению (рис. 6).

1. Пусть a – абсцисса
точки касания, лежащей на графике функции y = x2
+ x + 1.
2. f(a) = a2 + a + 1.
3. f ‘(a) = 2a + 1.
4. y = a2 + a + 1 + (2a + 1)(x – a) = (2a + 1)x + 1 – a2.

1. Пусть c – абсцисса
точки касания, лежащей на графике функции  
2.
3. f ‘(c) = c.
4. 

Так как касательные общие, то

Итак, y = x + 1 и y = – 3x – 3
– общие касательные.

Основная цель рассмотренных
задач – подготовить учащихся к
самостоятельному распознаванию типа ключевой
задачи при решении более сложных задач,
требующих определенных исследовательских
умений (умения анализировать, сравнивать,
обобщать, выдвигать гипотезу и т. д.). К числу
таких задач можно отнести любую задачу, в которую
ключевая задача входит как составляющая.
Рассмотрим в качестве примера задачу (обратную
задаче 1) на нахождение функции по семейству ее
касательных.

3. При каких b и c прямые y = x и
y = – 2x являются касательными к графику функции
y = x2 + bx + c?

Решение.

Пусть t – абсцисса точки
касания прямой y = x с параболой y = x2 + bx + c; p
– абсцисса точки касания прямой y = – 2x с
параболой y = x2 + bx + c. Тогда уравнение
касательной y = x примет вид y = (2t + b)x + c – t2, а
уравнение касательной y = – 2x примет вид y = (2p +
b)x + c – p2.

Составим и решим систему
уравнений

Ответ:

 Задачи для
самостоятельного решения

1. Напишите уравнения
касательных, проведенных к графику функции y = 2x2
– 4x + 3 в точках пересечения графика с прямой y = x +
3.

Ответ: y = – 4x + 3, y = 6x – 9,5.

2. При каких значениях a
касательная, проведенная к графику функции y = x2
– ax в точке графика с абсциссой x0 = 1,
проходит через точку M(2; 3)?

Ответ: a = 0,5.

3. При каких значениях p
прямая y = px – 5 касается кривой y = 3x2 – 4x – 2?

Ответ: p1 = – 10, p2
= 2.

4. Найдите все общие точки
графика функции y = 3x – x3 и касательной,
проведенной к этому графику через точку P(0; 16).

Ответ: A(2; – 2), B(– 4; 52).

5. Найдите кратчайшее
расстояние между параболой y = x2 + 6x + 10 и
прямой

Ответ:

6. На кривой y = x2 – x + 1
найдите точку, в которой касательная к графику
параллельна прямой y – 3x + 1 = 0.

Ответ: M(2; 3).

7. Напишите уравнение
касательной к графику функции y = x2 + 2x –
| 4x |, которая касается его в двух точках.
Сделайте чертеж.

Ответ: y = 2x – 4.

8. Докажите, что прямая y = 2x
– 1 не пересекает кривую y = x4 + 3x2 + 2x.
Найдите расстояние между их ближайшими точками.

Ответ:

9. На параболе y = x2
взяты две точки с абсциссами x1 = 1, x2 = 3.
Через эти точки проведена секущая. В какой точке
параболы касательная к ней будет параллельна
проведенной секущей? Напишите уравнения секущей
и касательной.

Ответ: y = 4x – 3 – уравнение
секущей; y = 4x – 4 – уравнение касательной.

10. Найдите угол q между касательными
к графику функции y = x3 – 4x2 + 3x + 1,
проведенными в точках с абсциссами 0 и 1.

Ответ: q = 45°.

11. В каких точках
касательная к графику функции образует с осью Ox угол в 135°?

Ответ: A(0; – 1), B(4; 3).

12. В точке A(1; 8) к кривой проведена
касательная. Найдите длину отрезка касательной,
заключенного между осями координат.

Ответ:

13. Напишите уравнение всех
общих касательных к графикам функций y = x2
x + 1 и y = 2x2 – x + 0,5.

Ответ: y = – 3x и y = x.

14. Найдите расстояние между
касательными к графику функции параллельными оси абсцисс.

Ответ:

15. Определите, под какими
углами парабола y = x2 + 2x – 8 пересекает ось
абсцисс.

Ответ: q1 = arctg 6, q2 = arctg (– 6).

16. На графике функции найдите все
точки, касательная в каждой из которых к этому
графику пересекает положительные полуоси
координат, отсекая от них равные отрезки.

Ответ: A(– 3; 11).

17. Прямая y = 2x + 7 и парабола y
= x2 – 1 пересекаются в точках M и N. Найдите
точку K пересечения прямых, касающихся параболы в
точках M и N.

Ответ: K(1; – 9).

18. При каких значениях b
прямая y = 9x + b является касательной к графику
функции y = x3 – 3x + 15?

Ответ: – 1; 31.

19. При каких значениях k
прямая y = kx – 10 имеет только одну общую точку с
графиком функции y = 2x2 + 3x – 2? Для найденных
значений k определите координаты точки.

Ответ: k1 = – 5, A(– 2;
0); k2 = 11, B(2; 12).

20. При каких значениях b
касательная, проведенная к графику функции y = bx3
– 2x2 – 4 в точке с абсциссой x0 = 2,
проходит через точку M(1; 8)?

Ответ: b = – 3.

21. Парабола с вершиной на
оси Ox касается прямой, проходящей через точки A(1;
2) и B(2; 4), в точке B. Найдите уравнение параболы.

Ответ:

22. При каком значении
коэффициента k парабола y = x2 + kx + 1 касается
оси Ox?

Ответ: k = д 2.

23. Найдите углы между
прямой y = x + 2 и кривой y = 2x2 + 4x – 3.

Ответ:

24. Определите, под какими
углами пересекаются графики функций y = 2x2 +
3x – 3 и y = x2 + 2x + 3.

Ответ:

25. При каком значении k угол
между кривыми y = x2 + 2x + k и y = x2 + 4x + 4
будет равен 45°?

Ответ: k = – 3.

26. Найдите все значения x0,
при каждом из которых касательные к графикам
функции y = 5cos 3x + 2 и y = 3cos 5x в точках в
абсциссой x0 параллельны.

Ответ:

27. Под каким углом видна
окружность x2 + y2 = 16 из точки (8; 0)?

Ответ:

28. Найдите геометрическое
место точек, из которых парабола y = x2 видна
под прямым углом?

Ответ: прямая

29. Найдите расстояние между
касательными к графику функции образующими с
положительным направлением оси Ox угол 45°.

Ответ:

30. Найдите геометрическое
место вершин всех парабол вида y = x2 + ax + b,
касающихся прямой y = 4x – 1.

Ответ: прямая y = 4x + 3.

Литература

1. Звавич Л.И., Шляпочник Л.Я.,
Чинкина М.В. Алгебра и начала анализа: 3600 задач
для школьников и поступающих в вузы. – М., Дрофа,
1999.
2. Мордкович А. Семинар четвертый для молодых
учителей. Тема «Приложения производной». – М.,
«Математика», № 21/94.
3. Формирование знаний и умений на основе
теории поэтапного усвоения умственных действий.
/ Под ред. П.Я. Гальперина, Н.Ф. Талызиной.
– М., МГУ, 1968.

TopList

Уравнение касательной к графику функции

п.1. Уравнение касательной

Рассмотрим кривую (y=f(x)).
Выберем на ней точку A с координатами ((x_0,y_0)), проведем касательную AB в этой точке.

Как было показано в §42 данного справочника, угловой коэффициент касательной равен производной от функции f в точке (x_0): $$ k=f'(x_0) $$ Уравнение прямой AB, проведенной через две точки: ((y_B-y_A)=k(x_B-x_A)).
Для (A(x_0,y_0), B(x,y)) получаем: begin (y-y_0)=k(x-x_0)\ y=k(x-x_0)+y_0\ y=f'(x_0)(x-x_0)+f(x_0) end

Чтобы записать уравнение касательной с угловым коэффициентом в виде (y=kx+b), нужно раскрыть скобки и привести подобные: $$ y=f'(x_0)(x-x_0)+f(x_0)=underbrace_<=k>x+underbrace_ <=b>$$

п.2. Алгоритм построения касательной

На входе: уравнение кривой (y=f(x)), абсцисса точки касания (x_0).
Шаг 1. Найти значение функции в точке касания (f(x_0))
Шаг 2. Найти общее уравнение производной (f’ (x))
Шаг 3. Найти значение производной в точке касания (f'(x_0 ))
Шаг 4. Записать уравнение касательной (y=f’ (x_0)(x-x_0)+f(x_0)), привести его к виду (y=kx+b)
На выходе: уравнение касательной в виде (y=kx+b)

Пусть (f(x)=x^2+3).
Найдем касательную к этой параболе в точке (x_0=1).

(f(x_0)=1^2+3=4 )
(f'(x)=2x )
(f'(x_0)=2cdot 1=2)
Уравнение касательной: $$ y=2(x-1)+4=2x-2+4=2x+2 $$ Ответ: (y=2x+2)

п.3. Вертикальная касательная

Не путайте вертикальные касательные с вертикальными асимптотами.
Вертикальная асимптота проходит через точку разрыва 2-го рода (x_0notin D), в которой функция не определена и производная не существует. График функции приближается к асимптоте на бесконечности, но у них никогда не бывает общих точек.
А вертикальная касательная проходит через точку (x_0in D), входящую в область определения. График функции и касательная имеют одну общую точку ((x_0,y_0)).

Вертикальные касательные характерны для радикалов вида (y=sqrt[n]).

Пусть (f(x)=sqrt[5]+1).
Найдем касательную к этой кривой в точке (x_0=1).

(f(x_0)=sqrt[5]<1-1>+1=1)
(f'(x)=frac15(x-1)^<frac15-1>+0=frac15(x-1)^<-frac45>=frac<1><5(x-1)^<frac45>> )
(f'(x_0)=frac<1><5(1-1)^<frac45>>=frac10=+infty)
В точке (x_0) проходит вертикальная касательная.
Её уравнение: (x=1)
Ответ: (y=2x+2)

п.4. Примеры

Пример 1. Для функции (f(x)=2x^2+4x)
a) напишите уравнения касательных, проведенных к графику функции в точках его пересечения с осью OX.

Находим точки пересечения, решаем уравнение: $$ 2x^2+4x=0Rightarrow 2x(x+2)=0Rightarrow left[ begin x=0\ x=-2 end right. $$ Две точки на оси: (0;0) и (-2;0).
Касательная в точке (x_0=0): begin f(x_0)=0, f'(x)=4x+4\ f'(x_0)=4cdot 0+4=4\ y=4(x-0)+0=4x end Касательная в точке (x_0=-2): begin f(x_0)=0, f'(x)=4x+4\ f'(x_0)=4cdot (-2)+4=-4\ y=-4(x+2)+0=-4x-8 end

б) Найдите, в какой точке касательная образует с положительным направлением оси OX угол 45°. Напишите уравнение этой касательной.

Общее уравнение касательной: (f'(x)=4x+4)
По условию (f'(x_0)=tgalpha=tg45^circ=1)
Решаем уравнение: $$ 4x_0+4=1Rightarrow 4x_0=-3Rightarrow x_0=-frac34 $$ Точка касания (x_0=-frac34) begin f(x_0)=2cdotleft(-frac34right)^2+4cdotleft(-frac34right)=frac98-3=-frac<15> <8>end Уравнение касательной: begin y=1cdotleft(x+frac34right)-frac<15><8>=x-frac98 end

в) найдите, в какой точке касательная будет параллельна прямой (2x+y-6=0). Напишите уравнение этой касательной.

Найдем угловой коэффициент заданной прямой: (y=-2x+6Rightarrow k=-2).
Касательная должна быть параллельной, значит, её угловой коэффициент тоже (k=-2). Получаем уравнение: begin f'(x_0)=-2\ 4x_0+4=-2Rightarrow 4x_0=-6Rightarrow x_0=-frac32 end Точка касания (x_0=-frac32) begin f(x_0)=2cdotleft(-frac32right)^2+4cdotleft(-frac32right)=\ =frac92-6=-frac32 end Уравнение касательной: begin y=-2cdotleft(x+frac32right)-frac32=-2x-frac92 end Или, в каноническом виде: begin 2x+y+frac92=0 end

г) в какой точке функции можно провести горизонтальную касательную? Напишите уравнение этой касательной.

У горизонтальной прямой (k=0).
Получаем уравнение: (f'(x_0)=0). begin 4x_0+4=0Rightarrow 4x_0=-4Rightarrow x_0=-1 end Точка касания (x_0=-1) begin f(x_0)=2cdot(-1)^2+4cdot(-1)=-2 end Уравнение касательной: begin y=0cdot(x+1)-2=-2 end

Ответ: а) (y=4x) и (y=-4x-8); б) (y=x-frac98); в) (2x+y+frac92=0); г) (y=-2)

Пример 3*. Найдите точку, в которой касательная к графику функции (f(x)=frac-x) перпендикулярна прямой (y=11x+3). Напишите уравнение этой касательной.

Угловой коэффициент данной прямой (k_1=11).
Угловой коэффициент перпендикулярной прямой (k_2=-frac<1>=-frac<1><11>) begin f'(x)=left(fracright)’-x’=frac<2x(x+3)-(x^2+2)cdot 1><(x+3)^2>-1=frac<2x^2+6x-x^2-2-(x+3)^2><(x+3)^2>=\ =frac<(x+3)^2>=- frac<11> <(x+3)^2>end В точке касания: begin f'(x_0)=k_2Rightarrow=-frac<11><(x+3)^2>=-frac<1><11>Rightarrow (x+3)^2=121Rightarrow (x+3)^2-11^2=0Rightarrow\ Rightarrow (x+14)(x+8)=0Rightarrow left[ begin x=-14\ x=8 end right. end
Уравнение касательной при (x_0=-14) begin f(x_0)=frac<(-14)^2+2><-14+3>+14=frac<198><-11>+14=-18+14=-4\ y=-frac<1><11>(x+14)-4=-frac <11>end Уравнение касательной при (x_0=8) begin f(x_0)=frac<8^2+2><8+3>-8=frac<66><11>-8=-2\ y=-frac<1><11>(x-8)-2=-frac <11>end
Ответ: точка касания (-14;-4), уравнение (y=-frac<11>)
и точка касания (8;-2), уравнение (-frac<11>)

Пример 4*. Найдите уравнения общих касательных к параболам (y=x^2-5x+6) и (y=x^2+x+1). Укажите точки касания.

Найдем производные функций: begin f_1′(x)=2x-5, f_2′(x)=2x+1 end Пусть a – абсцисса точки касания для первой параболы, b — для второй.
Запишем уравнения касательных (g_1(x)) и (g_2(x)) через эти переменные. begin g_1(x)=f_1′(a)(x-a)+f_1(a)=(2a-5)(x-a)+a^2-5a+6=\ =(2a-5)x-2a^2+5a+a^2-5a+6=(2a-5)x+(6-a^2)\ \ g_2(x)=f_2′(b)(x-b)+f_2(b)=(2b+1)(x-b)+b^2+b+1=\ =(2b+1)x-2b^2-b+b^2+b+1=(2b+1)x+(1-b^2) end Для общей касательной должны быть равны угловые коэффициенты и свободные члены. Получаем систему уравнений: begin begin 2a-5=2b+1\ 6-a^2=1-b^2 end Rightarrow begin 2(a-b)=6\ a^2-b^2=5 end Rightarrow begin a-b=3\ (a-b)(a+b)=5 end Rightarrow begin a-b=3\ a+b=frac53 end Rightarrow \ Rightarrow begin 2a=3+frac53\ 2b=frac53-3 end Rightarrow begin a=frac73\ b=-frac23 end end Находим угловой коэффициент и свободный член из любого из двух уравнений касательных: $$ k=2a-5=2cdotfrac73-5=-frac13, b=6-a^2=6-frac<49><9>=frac59 $$ Уравнение общей касательной: $$ y=-frac x3+frac59 $$
Точки касания: begin a=frac73, f_1(a)=left(frac73right)^2-5cdotfrac73+6=frac<49><9>-frac<35><3>+6=frac<49-105+54><9>=-frac29\ b=-frac23, f_2(b)=left(-frac23right)^2-frac23+1=frac49-frac23+1frac<4-6+9><9>=frac79 end
Ответ: касательная (y=-frac x3+frac59); точки касания (left(frac73;-frac29right)) и (left(-frac23;frac79right))

Пример 5*. Докажите, что кривая (y=x^4+3x^2+2x) не пересекается с прямой (y=2x-1), и найдите расстояние между их ближайшими точками.

Решим уравнение: (x^4+3x^2+2x=2x-1) begin x^4+3x^2+1=0Rightarrow D=3^2-4=5Rightarrow x^2=frac<-3pmsqrt<5>> <2>end Оба корня отрицательные, а квадрат не может быть отрицательным числом.
Значит, (xinvarnothing) — решений нет, кривая и прямая не пересекаются.
Что и требовалось доказать.

Чтобы найти расстояние, необходимо построить касательную к кривой с тем же угловым коэффициентом (k=2), то и y данной прямой. Тогда искомым расстоянием будет расстояние от точки касания до прямой (y=2x-1).
Строим уравнение касательной. По условию: (f'(x)=4x^3+6x+2=2) begin 4x^3+6x=0Rightarrow 2x(2x^2+3)=0Rightarrow left[ begin x=0\ 2x^2+3=0 end right. Rightarrow left[ begin x=0\ x^2=-frac32 end right. Rightarrow left[ begin x=0\ xinvarnothing end right. Rightarrow x=0 end Точка касания (x_0=0, y_0=0^4+3cdot 0^2+2cdot 0=0).
Уравнение касательной: (y=2(x-0)+0=2x)

Ищем расстояние между двумя параллельными прямыми:
(y=2x) и (y=2x-1).
Перпендикуляр из точки (0;0) на прямую (y=2x-1) имеет угловой коэффициент (k=-frac12), его уравнение: (y=-frac12 x+b). Т.к. точка (0;0) принадлежит этому перпендикуляру, он проходит через начало координат и (b=0).

Уравнение перпендикуляра: (y=-frac x2).
Находим точку пересечения прямой (y=2x-1) и перпендикуляра (y=-frac x2): begin 2x-1=-frac x2Rightarrow 2,5x=1Rightarrow x=0,4; y=-frac<0,4><2>=-0,2 end Точка пересечения A(0,4;-0,2).
Находим расстояние (OA=sqrt<0,4^2+(-0,2)^2>=0,2sqrt<2^2+1^2>=frac<sqrt<5>><5>)
Ответ: (frac<sqrt<5>><5>)

Касательная к графику функции в точке. Уравнение касательной. Геометрический смысл производной

Статья дает подробное разъяснение определений, геометрического смысла производной с графическими обозначениями. Будет рассмотрено уравнение касательной прямой с приведением примеров, найдено уравнения касательной к кривым 2 порядка.

Определения и понятия

Угол наклона прямой y = k x + b называется угол α , который отсчитывается от положительного направления оси о х к прямой y = k x + b в положительном направлении.

На рисунке направление о х обозначается при помощи зеленой стрелки и в виде зеленой дуги, а угол наклона при помощи красной дуги. Синяя линия относится к прямой.

Угловой коэффициент прямой y = k x + b называют числовым коэффициентом k .

Угловой коэффициент равняется тангенсу наклона прямой, иначе говоря k = t g α .

  • Угол наклона прямой равняется 0 только при параллельности о х и угловом коэффициенте, равному нулю, потому как тангенс нуля равен 0 . Значит, вид уравнения будет y = b .
  • Если угол наклона прямой y = k x + b острый, тогда выполняются условия 0 α π 2 или 0 ° α 90 ° . Отсюда имеем, что значение углового коэффициента k считается положительным числом, потому как значение тангенс удовлетворяет условию t g α > 0 , причем имеется возрастание графика.
  • Если α = π 2 , тогда расположение прямой перпендикулярно о х . Равенство задается при помощи равенства x = c со значением с , являющимся действительным числом.
  • Если угол наклона прямой y = k x + b тупой, то соответствует условиям π 2 α π или 90 ° α 180 ° , значение углового коэффициента k принимает отрицательное значение, а график убывает.

Определение 3

Секущей называют прямую, которая проходит через 2 точки функции f ( x ) . Иначе говоря, секущая – это прямая, которая проводится через любые две точки графика заданной функции.

По рисунку видно, что А В является секущей, а f ( x ) – черная кривая, α — красная дуга, означающая угол наклона секущей.

Когда угловой коэффициент прямой равняется тангенсу угла наклона, то видно, что тангенс из прямоугольного треугольника А В С можно найти по отношению противолежащего катета к прилежащему.

Получаем формулу для нахождения секущей вида:

k = t g α = B C A C = f ( x B ) — f x A x B — x A , где абсциссами точек А и В являются значения x A , x B , а f ( x A ) , f ( x B ) — это значения функции в этих точках.

Очевидно, что угловой коэффициент секущей определен при помощи равенства k = f ( x B ) — f ( x A ) x B — x A или k = f ( x A ) — f ( x B ) x A — x B , причем уравнение необходимо записать как y = f ( x B ) — f ( x A ) x B — x A · x — x A + f ( x A ) или
y = f ( x A ) — f ( x B ) x A — x B · x — x B + f ( x B ) .

Секущая делит график визуально на 3 части: слева от точки А , от А до В , справа от В . На располагаемом ниже рисунке видно, что имеются три секущие, которые считаются совпадающими, то есть задаются при помощи аналогичного уравнения.

По определению видно, что прямая и ее секущая в данном случае совпадают.

Секущая может множественно раз пересекать график заданной функции. Если имеется уравнение вида у = 0 для секущей, тогда количество точек пересечения с синусоидой бесконечно.

Касательная к графику функции f ( x ) в точке x 0 ; f ( x 0 ) называется прямая, проходящая через заданную точку x 0 ; f ( x 0 ) , с наличием отрезка, который имеет множество значений х , близких к x 0 .

Рассмотрим подробно на ниже приведенном примере. Тогда видно, что прямая, заданная функцией y = x + 1 , считается касательной к y = 2 x в точке с координатами ( 1 ; 2 ) . Для наглядности, необходимо рассмотреть графики с приближенными к ( 1 ; 2 ) значениями. Функция y = 2 x обозначена черным цветом, синяя линия – касательная, красная точка – точка пересечения.

Очевидно, что y = 2 x сливается с прямой у = х + 1 .

Для определения касательной следует рассмотреть поведение касательной А В при бесконечном приближении точки В к точке А . Для наглядности приведем рисунок.

Секущая А В , обозначенная при помощи синей линии, стремится к положению самой касательной, а угол наклона секущей α начнет стремиться к углу наклона самой касательной α x .

Касательной к графику функции y = f ( x ) в точке А считается предельное положение секущей А В при В стремящейся к А , то есть B → A .

Теперь перейдем к рассмотрению геометрического смысла производной функции в точке.

Геометрический смысл производной функции в точке

Перейдем к рассмотрению секущей А В для функции f ( x ) , где А и В с координатами x 0 , f ( x 0 ) и x 0 + ∆ x , f ( x 0 + ∆ x ) , а ∆ x обозначаем как приращение аргумента. Теперь функция примет вид ∆ y = ∆ f ( x ) = f ( x 0 + ∆ x ) — f ( ∆ x ) . Для наглядности приведем в пример рисунок.

Рассмотрим полученный прямоугольный треугольник А В С . Используем определение тангенса для решения, то есть получим отношение ∆ y ∆ x = t g α . Из определения касательной следует, что lim ∆ x → 0 ∆ y ∆ x = t g α x . По правилу производной в точке имеем, что производную f ( x ) в точке x 0 называют пределом отношений приращения функции к приращению аргумента, где ∆ x → 0 , тогда обозначим как f ( x 0 ) = lim ∆ x → 0 ∆ y ∆ x .

Отсюда следует, что f ‘ ( x 0 ) = lim ∆ x → 0 ∆ y ∆ x = t g α x = k x , где k x обозначают в качестве углового коэффициента касательной.

То есть получаем, что f ’ ( x ) может существовать в точке x 0 причем как и касательная к заданному графику функции в точке касания равной x 0 , f 0 ( x 0 ) , где значение углового коэффициента касательной в точке равняется производной в точке x 0 . Тогда получаем, что k x = f ‘ ( x 0 ) .

Геометрический смысл производной функции в точке в том, что дается понятие существования касательной к графику в этой же точке.

Уравнение касательной прямой

Чтобы записать уравнение любой прямой на плоскости, необходимо иметь угловой коэффициент с точкой, через которую она проходит. Его обозначение принимается как x 0 при пересечении.

Уравнение касательной к графику функции y = f ( x ) в точке x 0 , f 0 ( x 0 ) принимает вид y = f ‘ ( x 0 ) · x — x 0 + f ( x 0 ) .

Имеется в виду, что конечным значением производной f ‘ ( x 0 ) можно определить положение касательной, то есть вертикально при условии lim x → x 0 + 0 f ‘ ( x ) = ∞ и lim x → x 0 — 0 f ‘ ( x ) = ∞ или отсутствие вовсе при условии lim x → x 0 + 0 f ‘ ( x ) ≠ lim x → x 0 — 0 f ‘ ( x ) .

Расположение касательной зависит от значения ее углового коэффициента k x = f ‘ ( x 0 ) . При параллельности к оси о х получаем, что k k = 0 , при параллельности к о у — k x = ∞ , причем вид уравнения касательной x = x 0 возрастает при k x > 0 , убывает при k x 0 .

Произвести составление уравнения касательной к графику функции y = e x + 1 + x 3 3 — 6 — 3 3 x — 17 — 3 3 в точке с координатами ( 1 ; 3 ) с определением угла наклона.

Решение

По условию имеем, что функция определяется для всех действительных чисел. Получаем, что точка с координатами, заданными по условию, ( 1 ; 3 ) является точкой касания, тогда x 0 = — 1 , f ( x 0 ) = — 3 .

Необходимо найти производную в точке со значением — 1 . Получаем, что

y ‘ = e x + 1 + x 3 3 — 6 — 3 3 x — 17 — 3 3 ‘ = = e x + 1 ‘ + x 3 3 ‘ — 6 — 3 3 x ‘ — 17 — 3 3 ‘ = e x + 1 + x 2 — 6 — 3 3 y ‘ ( x 0 ) = y ‘ ( — 1 ) = e — 1 + 1 + — 1 2 — 6 — 3 3 = 3 3

Значение f ’ ( x ) в точке касания является угловым коэффициентом касательной, который равняется тангенсу наклона.

Тогда k x = t g α x = y ‘ ( x 0 ) = 3 3

Отсюда следует, что α x = a r c t g 3 3 = π 6

Ответ: уравнение касательной приобретает вид

y = f ‘ ( x 0 ) · x — x 0 + f ( x 0 ) y = 3 3 ( x + 1 ) — 3 y = 3 3 x — 9 — 3 3

Для наглядности приведем пример в графической иллюстрации.

Черный цвет используется для графика исходной функции, синий цвет – изображение касательной, красная точка – точка касания. Рисунок, располагаемый справа, показывает в увеличенном виде.

Выяснить наличие существования касательной к графику заданной функции
y = 3 · x — 1 5 + 1 в точке с координатами ( 1 ; 1 ) . Составить уравнение и определить угол наклона.

Решение

По условию имеем, что областью определения заданной функции считается множество всех действительных чисел.

Перейдем к нахождению производной

y ‘ = 3 · x — 1 5 + 1 ‘ = 3 · 1 5 · ( x — 1 ) 1 5 — 1 = 3 5 · 1 ( x — 1 ) 4 5

Если x 0 = 1 , тогда f ’ ( x ) не определена, но пределы записываются как lim x → 1 + 0 3 5 · 1 ( x — 1 ) 4 5 = 3 5 · 1 ( + 0 ) 4 5 = 3 5 · 1 + 0 = + ∞ и lim x → 1 — 0 3 5 · 1 ( x — 1 ) 4 5 = 3 5 · 1 ( — 0 ) 4 5 = 3 5 · 1 + 0 = + ∞ , что означает существование вертикальной касательной в точке ( 1 ; 1 ) .

Ответ: уравнение примет вид х = 1 , где угол наклона будет равен π 2 .

Для наглядности изобразим графически.

Найти точки графика функции y = 1 15 x + 2 3 — 4 5 x 2 — 16 5 x — 26 5 + 3 x + 2 , где

  1. Касательная не существует;
  2. Касательная располагается параллельно о х ;
  3. Касательная параллельна прямой y = 8 5 x + 4 .

Решение

Необходимо обратить внимание на область определения. По условию имеем, что функция определена на множестве всех действительных чисел. Раскрываем модуль и решаем систему с промежутками x ∈ — ∞ ; 2 и [ — 2 ; + ∞ ) . Получаем, что

y = — 1 15 x 3 + 18 x 2 + 105 x + 176 , x ∈ — ∞ ; — 2 1 15 x 3 — 6 x 2 + 9 x + 12 , x ∈ [ — 2 ; + ∞ )

Необходимо продифференцировать функцию. Имеем, что

y ‘ = — 1 15 x 3 + 18 x 2 + 105 x + 176 ‘ , x ∈ — ∞ ; — 2 1 15 x 3 — 6 x 2 + 9 x + 12 ‘ , x ∈ [ — 2 ; + ∞ ) ⇔ y ‘ = — 1 5 ( x 2 + 12 x + 35 ) , x ∈ — ∞ ; — 2 1 5 x 2 — 4 x + 3 , x ∈ [ — 2 ; + ∞ )

Когда х = — 2 , тогда производная не существует, потому что односторонние пределы не равны в этой точке:

lim x → — 2 — 0 y ‘ ( x ) = lim x → — 2 — 0 — 1 5 ( x 2 + 12 x + 35 = — 1 5 ( — 2 ) 2 + 12 ( — 2 ) + 35 = — 3 lim x → — 2 + 0 y ‘ ( x ) = lim x → — 2 + 0 1 5 ( x 2 — 4 x + 3 ) = 1 5 — 2 2 — 4 — 2 + 3 = 3

Вычисляем значение функции в точке х = — 2 , где получаем, что

  1. y ( — 2 ) = 1 15 — 2 + 2 3 — 4 5 ( — 2 ) 2 — 16 5 ( — 2 ) — 26 5 + 3 — 2 + 2 = — 2 , то есть касательная в точке ( — 2 ; — 2 ) не будет существовать.
  2. Касательная параллельна о х , когда угловой коэффициент равняется нулю. Тогда k x = t g α x = f ‘ ( x 0 ) . То есть необходимо найти значения таких х , когда производная функции обращает ее в ноль. То есть значения f ’ ( x ) и будут являться точками касания, где касательная является параллельной о х .

Когда x ∈ — ∞ ; — 2 , тогда — 1 5 ( x 2 + 12 x + 35 ) = 0 , а при x ∈ ( — 2 ; + ∞ ) получаем 1 5 ( x 2 — 4 x + 3 ) = 0 .

— 1 5 ( x 2 + 12 x + 35 ) = 0 D = 12 2 — 4 · 35 = 144 — 140 = 4 x 1 = — 12 + 4 2 = — 5 ∈ — ∞ ; — 2 x 2 = — 12 — 4 2 = — 7 ∈ — ∞ ; — 2 1 5 ( x 2 — 4 x + 3 ) = 0 D = 4 2 — 4 · 3 = 4 x 3 = 4 — 4 2 = 1 ∈ — 2 ; + ∞ x 4 = 4 + 4 2 = 3 ∈ — 2 ; + ∞

Вычисляем соответствующие значения функции

y 1 = y — 5 = 1 15 — 5 + 2 3 — 4 5 — 5 2 — 16 5 — 5 — 26 5 + 3 — 5 + 2 = 8 5 y 2 = y ( — 7 ) = 1 15 — 7 + 2 3 — 4 5 ( — 7 ) 2 — 16 5 — 7 — 26 5 + 3 — 7 + 2 = 4 3 y 3 = y ( 1 ) = 1 15 1 + 2 3 — 4 5 · 1 2 — 16 5 · 1 — 26 5 + 3 1 + 2 = 8 5 y 4 = y ( 3 ) = 1 15 3 + 2 3 — 4 5 · 3 2 — 16 5 · 3 — 26 5 + 3 3 + 2 = 4 3

Отсюда — 5 ; 8 5 , — 4 ; 4 3 , 1 ; 8 5 , 3 ; 4 3 считаются искомыми точками графика функции.

Рассмотрим графическое изображение решения.

Черная линия – график функции, красные точки – точки касания.

  1. Когда прямые располагаются параллельно, то угловые коэффициенты равны. Тогда необходимо заняться поиском точек графика функции, где угловой коэффициент будет равняться значению 8 5 . Для этого нужно решить уравнение вида y ‘ ( x ) = 8 5 . Тогда, если x ∈ — ∞ ; — 2 , получаем, что — 1 5 ( x 2 + 12 x + 35 ) = 8 5 , а если x ∈ ( — 2 ; + ∞ ) , тогда 1 5 ( x 2 — 4 x + 3 ) = 8 5 .

Первое уравнение не имеет корней, так как дискриминант меньше нуля. Запишем, что

— 1 5 x 2 + 12 x + 35 = 8 5 x 2 + 12 x + 43 = 0 D = 12 2 — 4 · 43 = — 28 0

Другое уравнение имеет два действительных корня, тогда

1 5 ( x 2 — 4 x + 3 ) = 8 5 x 2 — 4 x — 5 = 0 D = 4 2 — 4 · ( — 5 ) = 36 x 1 = 4 — 36 2 = — 1 ∈ — 2 ; + ∞ x 2 = 4 + 36 2 = 5 ∈ — 2 ; + ∞

Перейдем к нахождению значений функции. Получаем, что

y 1 = y ( — 1 ) = 1 15 — 1 + 2 3 — 4 5 ( — 1 ) 2 — 16 5 ( — 1 ) — 26 5 + 3 — 1 + 2 = 4 15 y 2 = y ( 5 ) = 1 15 5 + 2 3 — 4 5 · 5 2 — 16 5 · 5 — 26 5 + 3 5 + 2 = 8 3

Точки со значениями — 1 ; 4 15 , 5 ; 8 3 являются точками, в которых касательные параллельны прямой y = 8 5 x + 4 .

Ответ: черная линия – график функции, красная линия – график y = 8 5 x + 4 , синяя линия – касательные в точках — 1 ; 4 15 , 5 ; 8 3 .

Возможно существование бесконечного количества касательных для заданных функций.

Написать уравнения всех имеющихся касательных функции y = 3 cos 3 2 x — π 4 — 1 3 , которые располагаются перпендикулярно прямой y = — 2 x + 1 2 .

Решение

Для составления уравнения касательной необходимо найти коэффициент и координаты точки касания, исходя из условия перпендикулярности прямых. Определение звучит так: произведение угловых коэффициентов, которые перпендикулярны прямым, равняется — 1 , то есть записывается как k x · k ⊥ = — 1 . Из условия имеем, что угловой коэффициент располагается перпендикулярно прямой и равняется k ⊥ = — 2 , тогда k x = — 1 k ⊥ = — 1 — 2 = 1 2 .

Теперь необходимо найти координаты точек касания. Нужно найти х , после чего его значение для заданной функции. Отметим, что из геометрического смысла производной в точке
x 0 получаем, что k x = y ‘ ( x 0 ) . Из данного равенства найдем значения х для точек касания.

y ‘ ( x 0 ) = 3 cos 3 2 x 0 — π 4 — 1 3 ‘ = 3 · — sin 3 2 x 0 — π 4 · 3 2 x 0 — π 4 ‘ = = — 3 · sin 3 2 x 0 — π 4 · 3 2 = — 9 2 · sin 3 2 x 0 — π 4 ⇒ k x = y ‘ ( x 0 ) ⇔ — 9 2 · sin 3 2 x 0 — π 4 = 1 2 ⇒ sin 3 2 x 0 — π 4 = — 1 9

Это тригонометрическое уравнение будет использовано для вычисления ординат точек касания.

3 2 x 0 — π 4 = a r c sin — 1 9 + 2 πk или 3 2 x 0 — π 4 = π — a r c sin — 1 9 + 2 πk

3 2 x 0 — π 4 = — a r c sin 1 9 + 2 πk или 3 2 x 0 — π 4 = π + a r c sin 1 9 + 2 πk

x 0 = 2 3 π 4 — a r c sin 1 9 + 2 πk или x 0 = 2 3 5 π 4 + a r c sin 1 9 + 2 πk , k ∈ Z

Z — множество целых чисел.

Найдены х точек касания. Теперь необходимо перейти к поиску значений у :

y 0 = 3 cos 3 2 x 0 — π 4 — 1 3

y 0 = 3 · 1 — sin 2 3 2 x 0 — π 4 — 1 3 или y 0 = 3 · — 1 — sin 2 3 2 x 0 — π 4 — 1 3

y 0 = 3 · 1 — — 1 9 2 — 1 3 или y 0 = 3 · — 1 — — 1 9 2 — 1 3

y 0 = 4 5 — 1 3 или y 0 = — 4 5 + 1 3

Отсюда получаем, что 2 3 π 4 — a r c sin 1 9 + 2 πk ; 4 5 — 1 3 , 2 3 5 π 4 + a r c sin 1 9 + 2 πk ; — 4 5 + 1 3 являются точками касания.

Ответ: необходимы уравнения запишутся как

y = 1 2 x — 2 3 π 4 — a r c sin 1 9 + 2 πk + 4 5 — 1 3 , y = 1 2 x — 2 3 5 π 4 + a r c sin 1 9 + 2 πk — 4 5 + 1 3 , k ∈ Z

Для наглядного изображения рассмотрим функцию и касательную на координатной прямой.

Рисунок показывает, что расположение функции идет на промежутке [ — 10 ; 10 ] , где черная прямя – график функции, синие линии – касательные, которые располагаются перпендикулярно заданной прямой вида y = — 2 x + 1 2 . Красные точки – это точки касания.

Касательная к окружности, эллипсу, гиперболе, параболе

Канонические уравнения кривых 2 порядка не являются однозначными функциями. Уравнения касательных для них составляются по известным схемам.

Касательная к окружности

Для задания окружности с центром в точке x c e n t e r ; y c e n t e r и радиусом R применяется формула x — x c e n t e r 2 + y — y c e n t e r 2 = R 2 .

Данное равенство может быть записано как объединение двух функций:

y = R 2 — x — x c e n t e r 2 + y c e n t e r y = — R 2 — x — x c e n t e r 2 + y c e n t e r

Первая функция располагается вверху, а вторая внизу, как показано на рисунке.

Для составления уравнения окружности в точке x 0 ; y 0 , которая располагается в верхней или нижней полуокружности, следует найти уравнение графика функции вида y = R 2 — x — x c e n t e r 2 + y c e n t e r или y = — R 2 — x — x c e n t e r 2 + y c e n t e r в указанной точке.

Когда в точках x c e n t e r ; y c e n t e r + R и x c e n t e r ; y c e n t e r — R касательные могут быть заданы уравнениями y = y c e n t e r + R и y = y c e n t e r — R , а в точках x c e n t e r + R ; y c e n t e r и
x c e n t e r — R ; y c e n t e r будут являться параллельными о у , тогда получим уравнения вида x = x c e n t e r + R и x = x c e n t e r — R .

Касательная к эллипсу

Когда эллипс имеет центр в точке x c e n t e r ; y c e n t e r с полуосями a и b , тогда он может быть задан при помощи уравнения x — x c e n t e r 2 a 2 + y — y c e n t e r 2 b 2 = 1 .

Эллипс и окружность могут быть обозначаться при помощи объединения двух функций, а именно: верхнего и нижнего полуэллипса. Тогда получаем, что

y = b a · a 2 — ( x — x c e n t e r ) 2 + y c e n t e r y = — b a · a 2 — ( x — x c e n t e r ) 2 + y c e n t e r

Если касательные располагаются на вершинах эллипса, тогда они параллельны о х или о у . Ниже для наглядности рассмотрим рисунок.

Написать уравнение касательной к эллипсу x — 3 2 4 + y — 5 2 25 = 1 в точках со значениями x равного х = 2 .

Решение

Необходимо найти точки касания, которые соответствуют значению х = 2 . Производим подстановку в имеющееся уравнение эллипса и получаем, что

x — 3 2 4 x = 2 + y — 5 2 25 = 1 1 4 + y — 5 2 25 = 1 ⇒ y — 5 2 = 3 4 · 25 ⇒ y = ± 5 3 2 + 5

Тогда 2 ; 5 3 2 + 5 и 2 ; — 5 3 2 + 5 являются точками касания, которые принадлежат верхнему и нижнему полуэллипсу.

Перейдем к нахождению и разрешению уравнения эллипса относительно y . Получим, что

x — 3 2 4 + y — 5 2 25 = 1 y — 5 2 25 = 1 — x — 3 2 4 ( y — 5 ) 2 = 25 · 1 — x — 3 2 4 y — 5 = ± 5 · 1 — x — 3 2 4 y = 5 ± 5 2 4 — x — 3 2

Очевидно, что верхний полуэллипс задается с помощью функции вида y = 5 + 5 2 4 — x — 3 2 , а нижний y = 5 — 5 2 4 — x — 3 2 .

Применим стандартный алгоритм для того, чтобы составить уравнение касательной к графику функции в точке. Запишем, что уравнение для первой касательной в точке 2 ; 5 3 2 + 5 будет иметь вид

y ‘ = 5 + 5 2 4 — x — 3 2 ‘ = 5 2 · 1 2 4 — ( x — 3 ) 2 · 4 — ( x — 3 ) 2 ‘ = = — 5 2 · x — 3 4 — ( x — 3 ) 2 ⇒ y ‘ ( x 0 ) = y ‘ ( 2 ) = — 5 2 · 2 — 3 4 — ( 2 — 3 ) 2 = 5 2 3 ⇒ y = y ‘ ( x 0 ) · x — x 0 + y 0 ⇔ y = 5 2 3 ( x — 2 ) + 5 3 2 + 5

Получаем, что уравнение второй касательной со значением в точке
2 ; — 5 3 2 + 5 принимает вид

y ‘ = 5 — 5 2 4 — ( x — 3 ) 2 ‘ = — 5 2 · 1 2 4 — ( x — 3 ) 2 · 4 — ( x — 3 ) 2 ‘ = = 5 2 · x — 3 4 — ( x — 3 ) 2 ⇒ y ‘ ( x 0 ) = y ‘ ( 2 ) = 5 2 · 2 — 3 4 — ( 2 — 3 ) 2 = — 5 2 3 ⇒ y = y ‘ ( x 0 ) · x — x 0 + y 0 ⇔ y = — 5 2 3 ( x — 2 ) — 5 3 2 + 5

Графически касательные обозначаются так:

Касательная к гиперболе

Когда гипербола имеет центр в точке x c e n t e r ; y c e n t e r и вершины x c e n t e r + α ; y c e n t e r и x c e n t e r — α ; y c e n t e r , имеет место задание неравенства x — x c e n t e r 2 α 2 — y — y c e n t e r 2 b 2 = 1 , если с вершинами x c e n t e r ; y c e n t e r + b и x c e n t e r ; y c e n t e r — b , тогда задается при помощи неравенства x — x c e n t e r 2 α 2 — y — y c e n t e r 2 b 2 = — 1 .

Гипербола может быть представлена в виде двух объединенных функций вида

y = b a · ( x — x c e n t e r ) 2 — a 2 + y c e n t e r y = — b a · ( x — x c e n t e r ) 2 — a 2 + y c e n t e r или y = b a · ( x — x c e n t e r ) 2 + a 2 + y c e n t e r y = — b a · ( x — x c e n t e r ) 2 + a 2 + y c e n t e r

В первом случае имеем, что касательные параллельны о у , а во втором параллельны о х .

Отсюда следует, что для того, чтобы найти уравнение касательной к гиперболе, необходимо выяснить, какой функции принадлежит точка касания. Чтобы определить это, необходимо произвести подстановку в уравнения и проверить их на тождественность.

Составить уравнение касательной к гиперболе x — 3 2 4 — y + 3 2 9 = 1 в точке 7 ; — 3 3 — 3 .

Решение

Необходимо преобразовать запись решения нахождения гиперболы при помощи 2 функций. Получим, что

x — 3 2 4 — y + 3 2 9 = 1 ⇒ y + 3 2 9 = x — 3 2 4 — 1 ⇒ y + 3 2 = 9 · x — 3 2 4 — 1 ⇒ y + 3 = 3 2 · x — 3 2 — 4 и л и y + 3 = — 3 2 · x — 3 2 — 4 ⇒ y = 3 2 · x — 3 2 — 4 — 3 y = — 3 2 · x — 3 2 — 4 — 3

Необходимо выявить, к какой функции принадлежит заданная точка с координатами 7 ; — 3 3 — 3 .

Очевидно, что для проверки первой функции необходимо y ( 7 ) = 3 2 · ( 7 — 3 ) 2 — 4 — 3 = 3 3 — 3 ≠ — 3 3 — 3 , тогда точка графику не принадлежит, так как равенство не выполняется.

Для второй функции имеем, что y ( 7 ) = — 3 2 · ( 7 — 3 ) 2 — 4 — 3 = — 3 3 — 3 ≠ — 3 3 — 3 , значит, точка принадлежит заданному графику. Отсюда следует найти угловой коэффициент.

y ‘ = — 3 2 · ( x — 3 ) 2 — 4 — 3 ‘ = — 3 2 · x — 3 ( x — 3 ) 2 — 4 ⇒ k x = y ‘ ( x 0 ) = — 3 2 · x 0 — 3 x 0 — 3 2 — 4 x 0 = 7 = — 3 2 · 7 — 3 7 — 3 2 — 4 = — 3

Ответ: уравнение касательной можно представить как

y = — 3 · x — 7 — 3 3 — 3 = — 3 · x + 4 3 — 3

Наглядно изображается так:

Касательная к параболе

Чтобы составить уравнение касательной к параболе y = a x 2 + b x + c в точке x 0 , y ( x 0 ) , необходимо использовать стандартный алгоритм, тогда уравнение примет вид y = y ‘ ( x 0 ) · x — x 0 + y ( x 0 ) . Такая касательная в вершине параллельна о х .

Следует задать параболу x = a y 2 + b y + c как объединение двух функций. Поэтому нужно разрешить уравнение относительно у . Получаем, что

x = a y 2 + b y + c ⇔ a y 2 + b y + c — x = 0 D = b 2 — 4 a ( c — x ) y = — b + b 2 — 4 a ( c — x ) 2 a y = — b — b 2 — 4 a ( c — x ) 2 a

Графически изобразим как:

Для выяснения принадлежности точки x 0 , y ( x 0 ) функции, нежно действовать по стандартному алгоритму. Такая касательная будет параллельна о у относительно параболы.

Написать уравнение касательной к графику x — 2 y 2 — 5 y + 3 , когда имеем угол наклона касательной 150 ° .

Решение

Начинаем решение с представления параболы в качестве двух функций. Получим, что

— 2 y 2 — 5 y + 3 — x = 0 D = ( — 5 ) 2 — 4 · ( — 2 ) · ( 3 — x ) = 49 — 8 x y = 5 + 49 — 8 x — 4 y = 5 — 49 — 8 x — 4

Значение углового коэффициента равняется значению производной в точке x 0 этой функции и равняется тангенсу угла наклона.

k x = y ‘ ( x 0 ) = t g α x = t g 150 ° = — 1 3

Отсюда определим значение х для точек касания.

Первая функция запишется как

y ‘ = 5 + 49 — 8 x — 4 ‘ = 1 49 — 8 x ⇒ y ‘ ( x 0 ) = 1 49 — 8 x 0 = — 1 3 ⇔ 49 — 8 x 0 = — 3

Очевидно, что действительных корней нет, так как получили отрицательное значение. Делаем вывод, что касательной с углом 150 ° для такой функции не существует.

Вторая функция запишется как

y ‘ = 5 — 49 — 8 x — 4 ‘ = — 1 49 — 8 x ⇒ y ‘ ( x 0 ) = — 1 49 — 8 x 0 = — 1 3 ⇔ 49 — 8 x 0 = — 3 x 0 = 23 4 ⇒ y ( x 0 ) = 5 — 49 — 8 · 23 4 — 4 = — 5 + 3 4

Имеем, что точки касания — 23 4 ; — 5 + 3 4 .

Ответ: уравнение касательной принимает вид

Решение задач по математике онлайн

//mailru,yandex,google,vkontakte,odnoklassniki,instagram,wargaming,facebook,twitter,liveid,steam,soundcloud,lastfm, // echo( ‘

Калькулятор онлайн.
Уравнение прямой касательной к графику функции в заданной точке

Эта математическая программа находит уравнение касательной к графику функции ( f(x) ) в заданной пользователем точке ( x_0 ).

Программа не только выводит уравнение касательной, но и отображает процесс решения задачи.

Этот калькулятор онлайн может быть полезен учащимся старших классов общеобразовательных школ при подготовке к контрольным работам и экзаменам, при проверке знаний перед ЕГЭ, родителям для контроля решения многих задач по математике и алгебре. А может быть вам слишком накладно нанимать репетитора или покупать новые учебники? Или вы просто хотите как можно быстрее сделать домашнее задание по математике или алгебре? В этом случае вы также можете воспользоваться нашими программами с подробным решением.

Таким образом вы можете проводить своё собственное обучение и/или обучение своих младших братьев или сестёр, при этом уровень образования в области решаемых задач повышается.

Статью из энциклопедии о касательной прямой вы можете посмотреть здесь (статья из Википедии).

Если вам нужно найти производную функции, то для этого у нас есть задача Найти производную.

Обязательно ознакомьтесь с правилами ввода функций. Это сэкономит ваше время и нервы.
Правила ввода функций >> Почему решение на английском языке? >>
Введите выражение функции ( f(x)) и число (x_0) — абсциссу точки в которой нужно построить касательную Найти уравнение касательной

Немного теории.

Угловой коэффициент прямой

Напомним, что графиком линейной функции ( y=kx+b) является прямая. Число (k=tg alpha ) называют угловым коэффициентом прямой, а угол ( alpha ) — углом между этой прямой и осью Ox

Уравнение касательной к графику функции

Если точка М(а; f(a)) принадлежит графику функции у = f(x) и если в этой точке к графику функции можно провести касательную, не перпендикулярную оси абсцисс, то из геометрического смысла производной следует, что угловой коэффициент касательной равен f'(a). Далее мы выработаем алгоритм составления уравнения касательной к графику любой функции.

Пусть даны функция у = f(x) и точка М(а; f(a)) на графике этой функции; пусть известно, что существует f'(a). Составим уравнение касательной к графику заданной функции в заданной точке. Это уравнение, как уравнение любой прямой, не параллельной оси ординат, имеет вид y = kx + b, поэтому задача состоит в нахождении значений коэффициентов k и b.

С угловым коэффициентом k все понятно: известно, что k = f'(a). Для вычисления значения b воспользуемся тем, что искомая прямая проходит через точку М(а; f(a)). Это значит, что если подставить координаты точки М в уравнение прямой, получим верное равенство: (f(a)=ka+b ), т.е. ( b = f(a) — ka ).

Осталось подставить найденные значения коэффициентов k и b в уравнение прямой:

Нами получено уравнение касательной к графику функции ( y = f(x) ) в точке ( x=a ).

Алгоритм нахождения уравнения касательной к графику функции ( y=f(x) )
1. Обозначить абсциссу точки касания буквой ( a )
2. Вычислить ( f(a) )
3. Найти (f'(x) ) и вычислить (f'(a) )
4. Подставить найденные числа ( a, f(a), f'(a) ) в формулу ( y=f(a)+ f'(a)(x-a) )

источники:

http://zaochnik.com/spravochnik/matematika/proizvodnye/kasatelnaja-k-grafiku-funktsii-v-tochke/

http://www.math-solution.ru/math-task/equation-tangent

п.1. Уравнение касательной

Рассмотрим кривую (y=f(x)).
Выберем на ней точку A с координатами ((x_0,y_0)), проведем касательную AB в этой точке.
Уравнение касательной
Как было показано в §42 данного справочника, угловой коэффициент касательной равен производной от функции f в точке (x_0): $$ k=f'(x_0) $$ Уравнение прямой AB, проведенной через две точки: ((y_B-y_A)=k(x_B-x_A)).
Для (A(x_0,y_0), B(x,y)) получаем: begin{gather*} (y-y_0)=k(x-x_0)\ y=k(x-x_0)+y_0\ y=f'(x_0)(x-x_0)+f(x_0) end{gather*}

Уравнение касательной к кривой (y=f(x)) в точке (x_0) имеет вид: $$ y=f'(x_0)(x-x_0)+f(x_0) $$ при условии, что производная (f'(x_0)=aneinfty) — существует и конечна.

Чтобы записать уравнение касательной с угловым коэффициентом в виде (y=kx+b), нужно раскрыть скобки и привести подобные: $$ y=f'(x_0)(x-x_0)+f(x_0)=underbrace{f'(x_0)}_{=k}x+underbrace{f(x_0)-f'(x_0)cdot x_0}_{=b} $$

Уравнение касательной с угловым коэффициентом: begin{gather*} y=kx+b\ k=f'(x_0), b=f(x_0)-f'(x_0)cdot x_0 end{gather*}

п.2. Алгоритм построения касательной

На входе: уравнение кривой (y=f(x)), абсцисса точки касания (x_0).
Шаг 1. Найти значение функции в точке касания (f(x_0))
Шаг 2. Найти общее уравнение производной (f’ (x))
Шаг 3. Найти значение производной в точке касания (f'(x_0 ))
Шаг 4. Записать уравнение касательной (y=f’ (x_0)(x-x_0)+f(x_0)), привести его к виду (y=kx+b)
На выходе: уравнение касательной в виде (y=kx+b)

Например:

Алгоритм построения касательной Пусть (f(x)=x^2+3).
Найдем касательную к этой параболе в точке (x_0=1).

(f(x_0)=1^2+3=4 )
(f'(x)=2x )
(f'(x_0)=2cdot 1=2)
Уравнение касательной: $$ y=2(x-1)+4=2x-2+4=2x+2 $$ Ответ: (y=2x+2)

п.3. Вертикальная касательная

В случае, если производная (f'(x_0)=pminfty) — существует, но бесконечна, в точке (x_0) проходит вертикальная касательная (x=x_0).

Внимание!

Не путайте вертикальные касательные с вертикальными асимптотами.
Вертикальная асимптота проходит через точку разрыва 2-го рода (x_0notin D), в которой функция не определена и производная не существует. График функции приближается к асимптоте на бесконечности, но у них никогда не бывает общих точек.
А вертикальная касательная проходит через точку (x_0in D), входящую в область определения. График функции и касательная имеют одну общую точку ((x_0,y_0)).

Вертикальные касательные характерны для радикалов вида (y=sqrt[n]{x}).

Например:

Вертикальная касательная Пусть (f(x)=sqrt[5]{x-1}+1).
Найдем касательную к этой кривой в точке (x_0=1).

(f(x_0)=sqrt[5]{1-1}+1=1)
(f'(x)=frac15(x-1)^{frac15-1}+0=frac15(x-1)^{-frac45}=frac{1}{5(x-1)^{frac45}} )
(f'(x_0)=frac{1}{5(1-1)^{frac45}}=frac10=+infty)
В точке (x_0) проходит вертикальная касательная.
Её уравнение: (x=1)
Ответ: (y=2x+2)

п.4. Примеры

Пример 1. Для функции (f(x)=2x^2+4x)
a) напишите уравнения касательных, проведенных к графику функции в точках его пересечения с осью OX.

Пример 1а Находим точки пересечения, решаем уравнение: $$ 2x^2+4x=0Rightarrow 2x(x+2)=0Rightarrow left[ begin{array}{l} x=0\ x=-2 end{array} right. $$ Две точки на оси: (0;0) и (-2;0).
Касательная в точке (x_0=0): begin{gather*} f(x_0)=0, f'(x)=4x+4\ f'(x_0)=4cdot 0+4=4\ y=4(x-0)+0=4x end{gather*} Касательная в точке (x_0=-2): begin{gather*} f(x_0)=0, f'(x)=4x+4\ f'(x_0)=4cdot (-2)+4=-4\ y=-4(x+2)+0=-4x-8 end{gather*}

б) Найдите, в какой точке касательная образует с положительным направлением оси OX угол 45°. Напишите уравнение этой касательной.

Пример 1б Общее уравнение касательной: (f'(x)=4x+4)
По условию (f'(x_0)=tgalpha=tg45^circ=1)
Решаем уравнение: $$ 4x_0+4=1Rightarrow 4x_0=-3Rightarrow x_0=-frac34 $$ Точка касания (x_0=-frac34) begin{gather*} f(x_0)=2cdotleft(-frac34right)^2+4cdotleft(-frac34right)=frac98-3=-frac{15}{8} end{gather*} Уравнение касательной: begin{gather*} y=1cdotleft(x+frac34right)-frac{15}{8}=x-frac98 end{gather*}

в) найдите, в какой точке касательная будет параллельна прямой (2x+y-6=0). Напишите уравнение этой касательной.

Пример 1в Найдем угловой коэффициент заданной прямой: (y=-2x+6Rightarrow k=-2).
Касательная должна быть параллельной, значит, её угловой коэффициент тоже (k=-2). Получаем уравнение: begin{gather*} f'(x_0)=-2\ 4x_0+4=-2Rightarrow 4x_0=-6Rightarrow x_0=-frac32 end{gather*} Точка касания (x_0=-frac32) begin{gather*} f(x_0)=2cdotleft(-frac32right)^2+4cdotleft(-frac32right)=\ =frac92-6=-frac32 end{gather*} Уравнение касательной: begin{gather*} y=-2cdotleft(x+frac32right)-frac32=-2x-frac92 end{gather*} Или, в каноническом виде: begin{gather*} 2x+y+frac92=0 end{gather*}

г) в какой точке функции можно провести горизонтальную касательную? Напишите уравнение этой касательной.

Пример 1г У горизонтальной прямой (k=0).
Получаем уравнение: (f'(x_0)=0). begin{gather*} 4x_0+4=0Rightarrow 4x_0=-4Rightarrow x_0=-1 end{gather*} Точка касания (x_0=-1) begin{gather*} f(x_0)=2cdot(-1)^2+4cdot(-1)=-2 end{gather*} Уравнение касательной: begin{gather*} y=0cdot(x+1)-2=-2 end{gather*}

Ответ: а) (y=4x) и (y=-4x-8); б) (y=x-frac98); в) (2x+y+frac92=0); г) (y=-2)

Пример 2. Напишите уравнение касательной к графику функции в заданной точке:
a) ( f(x)=frac5x+frac x5, x_0=4 ) begin{gather*} f(x_0)=frac54+frac45=frac{25+16}{20}=frac{41}{20}\ f'(x)=left(frac5xright)’+left(frac x5right)’=-frac{5}{x^2}+frac15=frac{-25+x^2}{5x^2}=frac{x^2-25}{5x^2}\ f'(x_0)=frac{4^2-25}{5cdot 4^2}=-frac{9}{80} end{gather*} Уравнение касательной: $$ y=-frac{9}{80}(x-4)+frac{41}{20}=-frac{9}{80}x+frac{9}{20}+frac{41}{20}=-frac{9}{80}x+2,5 $$
б) ( f(x)=frac{x^2+5}{3-x}, x_0=2 ) begin{gather*} f(x_0)=frac{2^2+5}{3-2}=frac91=9\ f'(x)=frac{(x^2+5)'(3-x)-(x^2+5)(3-x)’}{(3-x)^2}=frac{2x(3-x)+(x^2+5)}{(3-x)^2}=\ =frac{6x-2x^2+x^2+5}{(3-x)^2}=frac{-x^2+6x+5}{(3-x)^2}\ f'(x_0)=frac{-2^2+6cdot 2+5}{(3-2)^2}=13 end{gather*} Уравнение касательной: $$ y=13(x-2)+9=13x-26+9=13x-17 $$

Пример 3*. Найдите точку, в которой касательная к графику функции (f(x)=frac{x^2+2}{x+3}-x) перпендикулярна прямой (y=11x+3). Напишите уравнение этой касательной.

Угловой коэффициент данной прямой (k_1=11).
Угловой коэффициент перпендикулярной прямой (k_2=-frac{1}{k_1}=-frac{1}{11}) begin{gather*} f'(x)=left(frac{x^2+2}{x+3}right)’-x’=frac{2x(x+3)-(x^2+2)cdot 1}{(x+3)^2}-1=frac{2x^2+6x-x^2-2-(x+3)^2}{(x+3)^2}=\ =frac{x^2+6x-2-x^2-6x-9}{(x+3)^2}=- frac{11}{(x+3)^2} end{gather*} В точке касания: begin{gather*} f'(x_0)=k_2Rightarrow=-frac{11}{(x+3)^2}=-frac{1}{11}Rightarrow (x+3)^2=121Rightarrow (x+3)^2-11^2=0Rightarrow\ Rightarrow (x+14)(x+8)=0Rightarrow left[ begin{array}{l} x=-14\ x=8 end{array} right. end{gather*} Пример 3
Уравнение касательной при (x_0=-14) begin{gather*} f(x_0)=frac{(-14)^2+2}{-14+3}+14=frac{198}{-11}+14=-18+14=-4\ y=-frac{1}{11}(x+14)-4=-frac{x+58}{11} end{gather*} Уравнение касательной при (x_0=8) begin{gather*} f(x_0)=frac{8^2+2}{8+3}-8=frac{66}{11}-8=-2\ y=-frac{1}{11}(x-8)-2=-frac{x+14}{11} end{gather*}
Ответ: точка касания (-14;-4), уравнение (y=-frac{x+58}{11})
и точка касания (8;-2), уравнение (-frac{x+14}{11})

Пример 4*. Найдите уравнения общих касательных к параболам (y=x^2-5x+6) и (y=x^2+x+1). Укажите точки касания.

Найдем производные функций: begin{gather*} f_1′(x)=2x-5, f_2′(x)=2x+1 end{gather*} Пусть a – абсцисса точки касания для первой параболы, b — для второй.
Запишем уравнения касательных (g_1(x)) и (g_2(x)) через эти переменные. begin{gather*} g_1(x)=f_1′(a)(x-a)+f_1(a)=(2a-5)(x-a)+a^2-5a+6=\ =(2a-5)x-2a^2+5a+a^2-5a+6=(2a-5)x+(6-a^2)\ \ g_2(x)=f_2′(b)(x-b)+f_2(b)=(2b+1)(x-b)+b^2+b+1=\ =(2b+1)x-2b^2-b+b^2+b+1=(2b+1)x+(1-b^2) end{gather*} Для общей касательной должны быть равны угловые коэффициенты и свободные члены. Получаем систему уравнений: begin{gather*} begin{cases} 2a-5=2b+1\ 6-a^2=1-b^2 end{cases} Rightarrow begin{cases} 2(a-b)=6\ a^2-b^2=5 end{cases} Rightarrow begin{cases} a-b=3\ (a-b)(a+b)=5 end{cases} Rightarrow begin{cases} a-b=3\ a+b=frac53 end{cases} Rightarrow \ Rightarrow begin{cases} 2a=3+frac53\ 2b=frac53-3 end{cases} Rightarrow begin{cases} a=frac73\ b=-frac23 end{cases} end{gather*} Находим угловой коэффициент и свободный член из любого из двух уравнений касательных: $$ k=2a-5=2cdotfrac73-5=-frac13, b=6-a^2=6-frac{49}{9}=frac59 $$ Уравнение общей касательной: $$ y=-frac x3+frac59 $$ Пример 4
Точки касания: begin{gather*} a=frac73, f_1(a)=left(frac73right)^2-5cdotfrac73+6=frac{49}{9}-frac{35}{3}+6=frac{49-105+54}{9}=-frac29\ b=-frac23, f_2(b)=left(-frac23right)^2-frac23+1=frac49-frac23+1frac{4-6+9}{9}=frac79 end{gather*}
Ответ: касательная (y=-frac x3+frac59); точки касания (left(frac73;-frac29right)) и (left(-frac23;frac79right))

Пример 5*. Докажите, что кривая (y=x^4+3x^2+2x) не пересекается с прямой (y=2x-1), и найдите расстояние между их ближайшими точками.

Решим уравнение: (x^4+3x^2+2x=2x-1) begin{gather*} x^4+3x^2+1=0Rightarrow D=3^2-4=5Rightarrow x^2=frac{-3pmsqrt{5}}{2} end{gather*} Оба корня отрицательные, а квадрат не может быть отрицательным числом.
Значит, (xinvarnothing) — решений нет, кривая и прямая не пересекаются.
Что и требовалось доказать.

Чтобы найти расстояние, необходимо построить касательную к кривой с тем же угловым коэффициентом (k=2), то и y данной прямой. Тогда искомым расстоянием будет расстояние от точки касания до прямой (y=2x-1).
Строим уравнение касательной. По условию: (f'(x)=4x^3+6x+2=2) begin{gather*} 4x^3+6x=0Rightarrow 2x(2x^2+3)=0Rightarrow left[ begin{array}{l} x=0\ 2x^2+3=0 end{array} right. Rightarrow left[ begin{array}{l} x=0\ x^2=-frac32 end{array} right. Rightarrow left[ begin{array}{l} x=0\ xinvarnothing end{array} right. Rightarrow x=0 end{gather*} Точка касания (x_0=0, y_0=0^4+3cdot 0^2+2cdot 0=0).
Уравнение касательной: (y=2(x-0)+0=2x)

Пример 5 Ищем расстояние между двумя параллельными прямыми:
(y=2x) и (y=2x-1).
Перпендикуляр из точки (0;0) на прямую (y=2x-1) имеет угловой коэффициент (k=-frac12), его уравнение: (y=-frac12 x+b). Т.к. точка (0;0) принадлежит этому перпендикуляру, он проходит через начало координат и (b=0).

Уравнение перпендикуляра: (y=-frac x2).
Находим точку пересечения прямой (y=2x-1) и перпендикуляра (y=-frac x2): begin{gather*} 2x-1=-frac x2Rightarrow 2,5x=1Rightarrow x=0,4; y=-frac{0,4}{2}=-0,2 end{gather*} Точка пересечения A(0,4;-0,2).
Находим расстояние (OA=sqrt{0,4^2+(-0,2)^2}=0,2sqrt{2^2+1^2}=frac{sqrt{5}}{5})
Ответ: (frac{sqrt{5}}{5})

Уравнение касательной и нормали к кривой

Уравнение касательной к кривой $ y=f(x) $ в точке $ M(x_0,y_0) $ имеет вид:

$$ y — y(x_0) = y'(x_0) (x — x_0) $$

Уравнение нормали к кривой $ y $ в точке $ M(x_0,y_0) $ имеет вид:

Нормаль к кривой — это перпендикуляр к касательной, проведенный через точку касания.

Примеры решений

Составить уравнение касательной и нормали к кривой в точке $ M(1;-1) $:

$$ x^2 + 2xy^2 + 3y^4 = 6 $$

Найдем производную, дифференцируя функцию $ y(x) $ по переменной $ x $:

Учитывая, что $ y^2 $ и $ y^4 $ сложные функции продолжаем:

$$ 2x + 2y^2 + 4xyy’ + 12y^3 y’ = 0 $$

Выражаем $ y’ $ из полученного уравнения:

$$ 4xyy’ + 12y^3 y’ = -2x — 2y^2 $$

Выносим $ y’ $ за скобки:

$$ y'(4xy + 12y^3) = -2x — 2y^2 $$

Делим обе части уравнения на выражение $ 4xy+12y^3 $:

Теперь вычисляем значение $ y’ $:

Зная, что $ y’ = frac $ и $ y(x_0) = y(1) = -1 $ составляем уравнения касательной и нормали к кривой в точке $ M(1;-1) $.

Получаем уравнение касательной:

Записываем в красивой форме:

Получаем уравнение нормали:

Раскрываем скобки и записываем в красивой форме, полученное уравнение:

Если не получается решить свою задачу, то присылайте её к нам. Мы предоставим подробное решение. Вы сможете ознакомиться с ходом вычисления и почерпнуть информацию. Это поможет своевременно получить зачёт у преподавателя!

Уравнение касательной к графику функции

Рассмотрим кривую (y=f(x)).
Выберем на ней точку A с координатами ((x_0,y_0)), проведем касательную AB в этой точке.
Уравнение касательной
Как было показано в §42 данного справочника, угловой коэффициент касательной равен производной от функции f в точке (x_0): $$ k=f'(x_0) $$ Уравнение прямой AB, проведенной через две точки: ((y_B-y_A)=k(x_B-x_A)).
Для (A(x_0,y_0), B(x,y)) получаем: begin (y-y_0)=k(x-x_0)\ y=k(x-x_0)+y_0\ y=f'(x_0)(x-x_0)+f(x_0) end

Чтобы записать уравнение касательной с угловым коэффициентом в виде (y=kx+b), нужно раскрыть скобки и привести подобные: $$ y=f'(x_0)(x-x_0)+f(x_0)=underbrace_x+underbrace_ $$

п.2. Алгоритм построения касательной

На входе: уравнение кривой (y=f(x)), абсцисса точки касания (x_0).
Шаг 1. Найти значение функции в точке касания (f(x_0))
Шаг 2. Найти общее уравнение производной (f’ (x))
Шаг 3. Найти значение производной в точке касания (f'(x_0 ))
Шаг 4. Записать уравнение касательной (y=f’ (x_0)(x-x_0)+f(x_0)), привести его к виду (y=kx+b)
На выходе: уравнение касательной в виде (y=kx+b)

п.3. Вертикальная касательная

Не путайте вертикальные касательные с вертикальными асимптотами.
Вертикальная асимптота проходит через точку разрыва 2-го рода (x_0notin D), в которой функция не определена и производная не существует. График функции приближается к асимптоте на бесконечности, но у них никогда не бывает общих точек.
А вертикальная касательная проходит через точку (x_0in D), входящую в область определения. График функции и касательная имеют одну общую точку ((x_0,y_0)).

Вертикальные касательные характерны для радикалов вида (y=sqrt[n]).

п.4. Примеры

Пример 1. Для функции (f(x)=2x^2+4x)
a) напишите уравнения касательных, проведенных к графику функции в точках его пересечения с осью OX.

Пример 1а Находим точки пересечения, решаем уравнение: $$ 2x^2+4x=0Rightarrow 2x(x+2)=0Rightarrow left[ beginx=0\ x=-2 end right. $$ Две точки на оси: (0;0) и (-2;0).
Касательная в точке (x_0=0): begin f(x_0)=0, f'(x)=4x+4\ f'(x_0)=4cdot 0+4=4\ y=4(x-0)+0=4x end Касательная в точке (x_0=-2): begin f(x_0)=0, f'(x)=4x+4\ f'(x_0)=4cdot (-2)+4=-4\ y=-4(x+2)+0=-4x-8 end

б) Найдите, в какой точке касательная образует с положительным направлением оси OX угол 45°. Напишите уравнение этой касательной.

Пример 1б Общее уравнение касательной: (f'(x)=4x+4)
По условию (f'(x_0)=tgalpha=tg45^circ=1)
Решаем уравнение: $$ 4x_0+4=1Rightarrow 4x_0=-3Rightarrow x_0=-frac34 $$ Точка касания (x_0=-frac34) begin f(x_0)=2cdotleft(-frac34right)^2+4cdotleft(-frac34right)=frac98-3=-frac end Уравнение касательной: begin y=1cdotleft(x+frac34right)-frac=x-frac98 end

в) найдите, в какой точке касательная будет параллельна прямой (2x+y-6=0). Напишите уравнение этой касательной.

Пример 1в Найдем угловой коэффициент заданной прямой: (y=-2x+6Rightarrow k=-2).
Касательная должна быть параллельной, значит, её угловой коэффициент тоже (k=-2). Получаем уравнение: begin f'(x_0)=-2\ 4x_0+4=-2Rightarrow 4x_0=-6Rightarrow x_0=-frac32 end Точка касания (x_0=-frac32) begin f(x_0)=2cdotleft(-frac32right)^2+4cdotleft(-frac32right)=\ =frac92-6=-frac32 end Уравнение касательной: begin y=-2cdotleft(x+frac32right)-frac32=-2x-frac92 end Или, в каноническом виде: begin 2x+y+frac92=0 end

г) в какой точке функции можно провести горизонтальную касательную? Напишите уравнение этой касательной.

Пример 1г У горизонтальной прямой (k=0).
Получаем уравнение: (f'(x_0)=0). begin 4x_0+4=0Rightarrow 4x_0=-4Rightarrow x_0=-1 end Точка касания (x_0=-1) begin f(x_0)=2cdot(-1)^2+4cdot(-1)=-2 end Уравнение касательной: begin y=0cdot(x+1)-2=-2 end

Ответ: а) (y=4x) и (y=-4x-8); б) (y=x-frac98); в) (2x+y+frac92=0); г) (y=-2)

Пример 3*. Найдите точку, в которой касательная к графику функции (f(x)=frac-x) перпендикулярна прямой (y=11x+3). Напишите уравнение этой касательной.

Угловой коэффициент данной прямой (k_1=11).
Угловой коэффициент перпендикулярной прямой (k_2=-frac=-frac) begin f'(x)=left(fracright)’-x’=frac-1=frac=\ =frac=- frac end В точке касания: begin f'(x_0)=k_2Rightarrow=-frac=-fracRightarrow (x+3)^2=121Rightarrow (x+3)^2-11^2=0Rightarrow\ Rightarrow (x+14)(x+8)=0Rightarrow left[ beginx=-14\ x=8 end right. end Пример 3
Уравнение касательной при (x_0=-14) begin f(x_0)=frac+14=frac+14=-18+14=-4\ y=-frac(x+14)-4=-fracend Уравнение касательной при (x_0=8) begin f(x_0)=frac-8=frac-8=-2\ y=-frac(x-8)-2=-fracend
Ответ: точка касания (-14;-4), уравнение (y=-frac)
и точка касания (8;-2), уравнение (-frac)

Пример 4*. Найдите уравнения общих касательных к параболам (y=x^2-5x+6) и (y=x^2+x+1). Укажите точки касания.

Найдем производные функций: begin f_1′(x)=2x-5, f_2′(x)=2x+1 end Пусть a – абсцисса точки касания для первой параболы, b — для второй.
Запишем уравнения касательных (g_1(x)) и (g_2(x)) через эти переменные. begin g_1(x)=f_1′(a)(x-a)+f_1(a)=(2a-5)(x-a)+a^2-5a+6=\ =(2a-5)x-2a^2+5a+a^2-5a+6=(2a-5)x+(6-a^2)\ \ g_2(x)=f_2′(b)(x-b)+f_2(b)=(2b+1)(x-b)+b^2+b+1=\ =(2b+1)x-2b^2-b+b^2+b+1=(2b+1)x+(1-b^2) end Для общей касательной должны быть равны угловые коэффициенты и свободные члены. Получаем систему уравнений: begin begin 2a-5=2b+1\ 6-a^2=1-b^2 end Rightarrow begin 2(a-b)=6\ a^2-b^2=5 end Rightarrow begin a-b=3\ (a-b)(a+b)=5 end Rightarrow begin a-b=3\ a+b=frac53 end Rightarrow \ Rightarrow begin 2a=3+frac53\ 2b=frac53-3 end Rightarrow begin a=frac73\ b=-frac23 end end Находим угловой коэффициент и свободный член из любого из двух уравнений касательных: $$ k=2a-5=2cdotfrac73-5=-frac13, b=6-a^2=6-frac=frac59 $$ Уравнение общей касательной: $$ y=-frac x3+frac59 $$ Пример 4
Точки касания: begin a=frac73, f_1(a)=left(frac73right)^2-5cdotfrac73+6=frac-frac+6=frac=-frac29\ b=-frac23, f_2(b)=left(-frac23right)^2-frac23+1=frac49-frac23+1frac=frac79 end
Ответ: касательная (y=-frac x3+frac59); точки касания (left(frac73;-frac29right)) и (left(-frac23;frac79right))

Пример 5*. Докажите, что кривая (y=x^4+3x^2+2x) не пересекается с прямой (y=2x-1), и найдите расстояние между их ближайшими точками.

Решим уравнение: (x^4+3x^2+2x=2x-1) begin x^4+3x^2+1=0Rightarrow D=3^2-4=5Rightarrow x^2=frac> end Оба корня отрицательные, а квадрат не может быть отрицательным числом.
Значит, (xinvarnothing) — решений нет, кривая и прямая не пересекаются.
Что и требовалось доказать.

Чтобы найти расстояние, необходимо построить касательную к кривой с тем же угловым коэффициентом (k=2), то и y данной прямой. Тогда искомым расстоянием будет расстояние от точки касания до прямой (y=2x-1).
Строим уравнение касательной. По условию: (f'(x)=4x^3+6x+2=2) begin 4x^3+6x=0Rightarrow 2x(2x^2+3)=0Rightarrow left[ beginx=0\ 2x^2+3=0 end right. Rightarrow left[ beginx=0\ x^2=-frac32 end right. Rightarrow left[ beginx=0\ xinvarnothing end right. Rightarrow x=0 end Точка касания (x_0=0, y_0=0^4+3cdot 0^2+2cdot 0=0).
Уравнение касательной: (y=2(x-0)+0=2x)

Пример 5 Ищем расстояние между двумя параллельными прямыми:
(y=2x) и (y=2x-1).
Перпендикуляр из точки (0;0) на прямую (y=2x-1) имеет угловой коэффициент (k=-frac12), его уравнение: (y=-frac12 x+b). Т.к. точка (0;0) принадлежит этому перпендикуляру, он проходит через начало координат и (b=0).

Уравнение перпендикуляра: (y=-frac x2).
Находим точку пересечения прямой (y=2x-1) и перпендикуляра (y=-frac x2): begin 2x-1=-frac x2Rightarrow 2,5x=1Rightarrow x=0,4; y=-frac=-0,2 end Точка пересечения A(0,4;-0,2).
Находим расстояние (OA=sqrt=0,2sqrt=frac>)
Ответ: (frac>)

Касательная и нормаль к графику функции

Касательная, нормаль, подкасательная TP и поднормаль PN к графику функции

Пусть на некотором интервале X задана функция . Нас интересуют геометрические характеристики графика этой функции в некоторой заданной точке при значении аргумента , где . Пусть функция имеет в производную, которую будем обозначать как . Тогда через точку мы можем провести касательную к графику. Тангенс угла α между осью абсцисс x и касательной равен производной функции в точке :
(1) .
А само уравнение касательной имеет вид:
(2) .
В аналитической геометрии тангенс угла между прямой и осью абсцисс называют угловым коэффициентом прямой. Таким образом производная равна угловому коэффициенту касательной в .
См. Геометрический смысл производной

Прямая, перпендикулярная касательной, проведенной через точку , называется нормалью к графику функции в этой точке. Уравнение нормали имеет вид:
(3) .
См. Уравнение прямой с угловым коэффициентом ⇓

Пусть две кривые и пересекаются в точке . Тогда угол φ между касательными к этим кривым в точке называется углом между кривыми. Он определяется по формуле:
(4) , где .
Отсюда .
при .
Вывод формулы ⇓

Определения

Здесь мы приводим определения, которые встречаются в литературе, и имеют отношение к касательной и нормали. Вывод формул приводится в примере 1 ⇓.

Определение касательной приводится здесь. Уравнение касательной:
.

Касательная, нормаль, подкасательная, поднормаль

Касательная TM0, нормаль M0N, подкасательная TP, поднормаль PN. Нормалью к графику функции в точке называется прямая, перпендикулярная касательной, проведенной через эту точку. Уравнение нормали:
.
Отрезком касательной называют отрезок между точкой пересечения касательной с осью абсцисс и точкой .
.
Отрезком нормали называют отрезок между точкой пересечения нормали с осью абсцисс и точкой .
.
Подкасательной называют отрезок между точкой пересечения касательной с осью абсцисс и проекции точки на эту ось.
.
Поднормалью называют отрезок между точкой пересечения нормали с осью абсцисс и проекции точки на эту ось.
.
Углом между кривыми в точке их пересечения называют угол между касательными к кривым, проведенных через точку .

Полезные формулы из аналитической геометрии

Далее приводятся некоторые сведения из аналитической геометрии, которые могут оказаться полезными при решении задач.

Уравнение прямой, проходящей через две заданные точки и :
.
Здесь – направляющий вектор прямой.

Умножив это уравнение на , получим уравнение прямой в другом виде:
.
Здесь – вектор нормали прямой. Тогда само уравнение означает равенство нулю скалярного произведения векторов и .

Уравнение прямой, проходящей через точку параллельно вектору имеет вид:
.
Вектор называется направляющим вектором данной прямой. Это уравнение можно написать в параметрическом виде, введя параметр t :

Уравнение прямой, проходящей через точку перпендикулярно вектору имеет вид:
.
Вектор называется вектором нормали данной прямой.

Уравнение прямой с угловым коэффициентом k , проходящей через точку :
.
Угол α между прямой и осью x определяется по формуле:
.
Если две прямые взаимно перпендикулярны, то их угловые коэффициенты и связаны соотношением:
.

Уравнение прямой в отрезках, пересекающей оси координат в точках :
.

Примеры решения задач

Все примеры Ниже рассмотрены примеры решений следующих задач.
1. Найти уравнения касательной и нормали к кривой в точке . Найти длины отрезков касательной, нормали, подкасательной и поднормали. Решение ⇓
2. Составить уравнения касательной и нормали к циссоиде, заданной в параметрическом виде
, проведенных в точке . Решение ⇓
3. Заданной в неявном виде . Решение ⇓
4. Найти угол между кривыми и Решение ⇓

Пример 1

Составить уравнения касательной и нормали к кривой в точке . Найти длины отрезков касательной, нормали, подкасательной и поднормали.

Находим значение функции при :
.

Находим производную:
.
Находим производную в точке :
;
.

Находим уравнение касательной по формуле (2):
;
;
;
– уравнение касательной.
Строим касательную на графике. Поскольку касательная – это прямая, то нам нужно знать положения двух ее точек, и провести через них прямую.
При ;
при .
Проводим касательную через точки и .

Касательная и нормаль к графику функции y=x^2 в точке (1;1)

Касательная и нормаль к графику функции y=x 2 в точке M0(1;1).

Найдем угол α между касательной и осью абсцисс по формуле (1):
.
Подставляем :
;
.

Находим уравнение нормали по формуле (3):
;
;
;
;
;
– уравнение нормали.
Строим нормаль по двум точкам.
При ;
при .
Проводим нормаль через точки и .

Находим длину отрезка касательной . Из прямоугольника имеем:
.
Поясним использованную формулу. Поскольку , то . Тогда
.
Подставляем :
.

Находим длину отрезка подкасательной . Из прямоугольника имеем:
.
Подставляем :
.

Находим длину отрезка нормали . Поскольку и , то треугольники и подобны. Тогда . Из прямоугольника имеем:
.
Подставляем :
.

Находим длину отрезка поднормали . Из прямоугольника имеем:
.

Примечание.
При выводе формул, можно сначала определить длины отрезков подкасательной и поднормали, а затем из прямоугольников, по теореме Пифагора, найти длины отрезков касательной и нормали:
;
.

Уравнение касательной: ; уравнение нормали: ;
длина отрезка касательной: ; длина отрезка нормали: ; длина подкасательной: ; длина поднормали: .

Пример 2

Составить уравнения касательной и нормали к циссоиде, заданной в параметрическом виде , проведенных в точке .

Находим значения переменных при .
;
.
Обозначим эту точку как .

Находим производные переменных x и y по параметру t .
;
;
;
;
.

Подставляя , находим производную y по x в точке .
.

Касательная и нормаль к циссоиде

Касательная и нормаль к циссоиде в точке (2;2).

Применяя формулу (2), находим уравнение касательной к циссоиде, проходящей через точку .
;
;
;
.

Применяя формулу (3), находим уравнение нормали к циссоиде в точке .
;
;
;
.

Уравнение касательной: .
Уравнение нормали: .

Пример 3

Составить уравнения касательной и нормали к циссоиде, заданной в неявном виде:
(П3) ,
проведенных в точке .

Для получения уравнение касательной и нормали, нам нужно знать значение производной функции в заданной точке. Функция (П3) задана неявно. Поэтому применяем правило дифференцирования неявной функции. Для этого дифференцируем (П3) по x , считая, что y является функцией от x .
;
;
;
.
Отсюда
.

Находим производную в заданной точке, подставляя .
;
.

Находим уравнение касательной по формуле (2).
;
;
;
.

Находим уравнение нормали по формуле (3).
;
;
;
.

Касательная и нормаль к циссоиде изображены на рисунке ⇑.

Уравнение касательной: .
Уравнение нормали: .

Пример 4

Найти угол между кривыми и .

Найдем множество точек пересечения кривых, решая систему уравнений.

Левые части равны. Приравниваем правые части и выполняем преобразования.
;
(П4) .
Поскольку функция строго монотонна, то уравнение (П4) имеет один корень:
.
При . Кривые пересекаются в единственной точке . Обозначим ее как , где .

Введем обозначения для функций, с помощью которых заданы кривые:
.
Найдем их производные.
;
.
Найдем значения производных в точке , подставляя .
;
.

Ниже приводятся графики функций ⇓ и вывод формулы угла между кривыми.

Вывод формулы для угла между кривыми

Изложим вывод формулы (4). Для иллюстрации используем только что рассмотренный пример ⇑, в котором .

Рассмотрим две кривые, заданные уравнениями и , и пересекающиеся в некоторой точке . Докажем, что угол между кривыми определяется по формуле (4):
, где .
Или ;
при .

Угол между кривыми равен наименьшему углу между касательными.

Угол между кривыми равен наименьшему углу между касательными.

Проведем касательные к графикам функций в точке . Углы, которые образуют касательные с осью x обозначим как и . За положительное направление выберем направление против часовой стрелки. На рисунке . Считаем, что значения углов принадлежат интервалам . Согласно геометрическому смыслу производной,
.

В аналитической геометрии принято, что угол φ между прямыми равен наименьшему значению угла между ними.
Если , то ;
если , то .
Таким образом величина угла φ между касательными может находиться только в пределах
(Ф2) .

На рисунке угол между лучами и больше 90°, а между лучами и – меньше. Поэтому .

При доказательстве мы будем использовать соотношение:
, которое выполняется при .
Тогда в силу (Ф2),
.
Случай мы рассмотрим отдельно.

1) Пусть .
Тогда угол между прямыми . И мы имеем:
.
В конце мы подставили (Ф1).

2) Пусть .
Тогда ; . Поэтому . Это можно записать так: . Также применим формулу: . В результате получаем:

.

Этот случай изображен на рисунке ⇑.

3) Пусть .
При этом касательные взаимно перпендикулярны, . В этом случае , что указано в (4).

Использованная литература:
П.Е. Данько, А.Г. Попов, Т.Я.Кожевникова. Высшая математика в упражнениях и задачах. Часть 1. Москва, Высшая школа, 1980.
Л.Д. Кудрявцев, А.Д. Кутасов, В.И. Чехлов, М.И. Шабунин. Сборник задач по математическому анализу. Том 1. Москва, Физматлит, 2003.

Задача 32812 Написать уравнение касательной к кривой.

Условие

Написать уравнение касательной к кривой: y=sqrt(x)-2, зная, что эта касательная перпендикулярна прямой 4x — у = 0.

Решение

Произведение угловых коэффициентов взаимно
перпендикулярных прямых равно (-1).

Прямая 4х-у=0 имеет угловой коэффициент k=4
Значит, угловой коэффициент касательной
k=-1/4
геометрический смысл производной в точке:
k(касательной)=f `(x_(o))

f(x)=sqrt(x) — 2
f `(x)= (sqrt(x) — 2)`=1/2sqrt(x)
f`(x_(o))=1/2sqrt(x_(o))

это уравнение не имеет решений.
и на рисунке видно, что нельзя провести касательную с
угловым коэффициентом (-1/4),т.е. под тупым углом к оси ох

и фокус в том, что из y=sqrt(x)-2⇒
sqrt(x)=y+2
⇒ х=(y+2)^2 — парабола
и касательная, удовлетворяющая условию
у=(-1/4)х — 3 проходит в точке с абсциссой х_(о)=4 к другой ветви параболы
y=-sqrt(x)-2

Поэтому либо опечатка в условии
и должно быть y=-sqrt(x)-2
либо.
Уравнение касательной:
y — f (x_(o)) =f ` (x_(o))* (x — x_(o))

f(x)= — sqrt(x) — 2
f `(x)= ( — sqrt(x) — 2)`=- 1/2sqrt(x)
f`(x_(o))= — 1/2sqrt(x_(o))

f(4)=-sqrt(4)-2=-4
y — (-4)= -(1/4)*(x — 4) — уравнение касательной
y=(-1/4)x -3

Касательная к графику функции в точке. Уравнение касательной. Геометрический смысл производной

Статья дает подробное разъяснение определений, геометрического смысла производной с графическими обозначениями. Будет рассмотрено уравнение касательной прямой с приведением примеров, найдено уравнения касательной к кривым 2 порядка.

Определения и понятия

Угол наклона прямой y = k x + b называется угол α , который отсчитывается от положительного направления оси о х к прямой y = k x + b в положительном направлении.

На рисунке направление о х обозначается при помощи зеленой стрелки и в виде зеленой дуги, а угол наклона при помощи красной дуги. Синяя линия относится к прямой.

Угловой коэффициент прямой y = k x + b называют числовым коэффициентом k .

Угловой коэффициент равняется тангенсу наклона прямой, иначе говоря k = t g α .

  • Угол наклона прямой равняется 0 только при параллельности о х и угловом коэффициенте, равному нулю, потому как тангенс нуля равен 0 . Значит, вид уравнения будет y = b .
  • Если угол наклона прямой y = k x + b острый, тогда выполняются условия 0 α π 2 или 0 ° α 90 ° . Отсюда имеем, что значение углового коэффициента k считается положительным числом, потому как значение тангенс удовлетворяет условию t g α > 0 , причем имеется возрастание графика.
  • Если α = π 2 , тогда расположение прямой перпендикулярно о х . Равенство задается при помощи равенства x = c со значением с , являющимся действительным числом.
  • Если угол наклона прямой y = k x + b тупой, то соответствует условиям π 2 α π или 90 ° α 180 ° , значение углового коэффициента k принимает отрицательное значение, а график убывает.

Определение 3

Секущей называют прямую, которая проходит через 2 точки функции f ( x ) . Иначе говоря, секущая – это прямая, которая проводится через любые две точки графика заданной функции.

По рисунку видно, что А В является секущей, а f ( x ) – черная кривая, α — красная дуга, означающая угол наклона секущей.

Когда угловой коэффициент прямой равняется тангенсу угла наклона, то видно, что тангенс из прямоугольного треугольника А В С можно найти по отношению противолежащего катета к прилежащему.

Получаем формулу для нахождения секущей вида:

k = t g α = B C A C = f ( x B ) — f x A x B — x A , где абсциссами точек А и В являются значения x A , x B , а f ( x A ) , f ( x B ) — это значения функции в этих точках.

Очевидно, что угловой коэффициент секущей определен при помощи равенства k = f ( x B ) — f ( x A ) x B — x A или k = f ( x A ) — f ( x B ) x A — x B , причем уравнение необходимо записать как y = f ( x B ) — f ( x A ) x B — x A · x — x A + f ( x A ) или
y = f ( x A ) — f ( x B ) x A — x B · x — x B + f ( x B ) .

Секущая делит график визуально на 3 части: слева от точки А , от А до В , справа от В . На располагаемом ниже рисунке видно, что имеются три секущие, которые считаются совпадающими, то есть задаются при помощи аналогичного уравнения.

По определению видно, что прямая и ее секущая в данном случае совпадают.

Секущая может множественно раз пересекать график заданной функции. Если имеется уравнение вида у = 0 для секущей, тогда количество точек пересечения с синусоидой бесконечно.

Касательная к графику функции f ( x ) в точке x 0 ; f ( x 0 ) называется прямая, проходящая через заданную точку x 0 ; f ( x 0 ) , с наличием отрезка, который имеет множество значений х , близких к x 0 .

Рассмотрим подробно на ниже приведенном примере. Тогда видно, что прямая, заданная функцией y = x + 1 , считается касательной к y = 2 x в точке с координатами ( 1 ; 2 ) . Для наглядности, необходимо рассмотреть графики с приближенными к ( 1 ; 2 ) значениями. Функция y = 2 x обозначена черным цветом, синяя линия – касательная, красная точка – точка пересечения.

Очевидно, что y = 2 x сливается с прямой у = х + 1 .

Для определения касательной следует рассмотреть поведение касательной А В при бесконечном приближении точки В к точке А . Для наглядности приведем рисунок.

Секущая А В , обозначенная при помощи синей линии, стремится к положению самой касательной, а угол наклона секущей α начнет стремиться к углу наклона самой касательной α x .

Касательной к графику функции y = f ( x ) в точке А считается предельное положение секущей А В при В стремящейся к А , то есть B → A .

Теперь перейдем к рассмотрению геометрического смысла производной функции в точке.

Геометрический смысл производной функции в точке

Перейдем к рассмотрению секущей А В для функции f ( x ) , где А и В с координатами x 0 , f ( x 0 ) и x 0 + ∆ x , f ( x 0 + ∆ x ) , а ∆ x обозначаем как приращение аргумента. Теперь функция примет вид ∆ y = ∆ f ( x ) = f ( x 0 + ∆ x ) — f ( ∆ x ) . Для наглядности приведем в пример рисунок.

Рассмотрим полученный прямоугольный треугольник А В С . Используем определение тангенса для решения, то есть получим отношение ∆ y ∆ x = t g α . Из определения касательной следует, что lim ∆ x → 0 ∆ y ∆ x = t g α x . По правилу производной в точке имеем, что производную f ( x ) в точке x 0 называют пределом отношений приращения функции к приращению аргумента, где ∆ x → 0 , тогда обозначим как f ( x 0 ) = lim ∆ x → 0 ∆ y ∆ x .

Отсюда следует, что f ‘ ( x 0 ) = lim ∆ x → 0 ∆ y ∆ x = t g α x = k x , где k x обозначают в качестве углового коэффициента касательной.

То есть получаем, что f ’ ( x ) может существовать в точке x 0 причем как и касательная к заданному графику функции в точке касания равной x 0 , f 0 ( x 0 ) , где значение углового коэффициента касательной в точке равняется производной в точке x 0 . Тогда получаем, что k x = f ‘ ( x 0 ) .

Геометрический смысл производной функции в точке в том, что дается понятие существования касательной к графику в этой же точке.

Уравнение касательной прямой

Чтобы записать уравнение любой прямой на плоскости, необходимо иметь угловой коэффициент с точкой, через которую она проходит. Его обозначение принимается как x 0 при пересечении.

Уравнение касательной к графику функции y = f ( x ) в точке x 0 , f 0 ( x 0 ) принимает вид y = f ‘ ( x 0 ) · x — x 0 + f ( x 0 ) .

Имеется в виду, что конечным значением производной f ‘ ( x 0 ) можно определить положение касательной, то есть вертикально при условии lim x → x 0 + 0 f ‘ ( x ) = ∞ и lim x → x 0 — 0 f ‘ ( x ) = ∞ или отсутствие вовсе при условии lim x → x 0 + 0 f ‘ ( x ) ≠ lim x → x 0 — 0 f ‘ ( x ) .

Расположение касательной зависит от значения ее углового коэффициента k x = f ‘ ( x 0 ) . При параллельности к оси о х получаем, что k k = 0 , при параллельности к о у — k x = ∞ , причем вид уравнения касательной x = x 0 возрастает при k x > 0 , убывает при k x 0 .

Произвести составление уравнения касательной к графику функции y = e x + 1 + x 3 3 — 6 — 3 3 x — 17 — 3 3 в точке с координатами ( 1 ; 3 ) с определением угла наклона.

Решение

По условию имеем, что функция определяется для всех действительных чисел. Получаем, что точка с координатами, заданными по условию, ( 1 ; 3 ) является точкой касания, тогда x 0 = — 1 , f ( x 0 ) = — 3 .

Необходимо найти производную в точке со значением — 1 . Получаем, что

y ‘ = e x + 1 + x 3 3 — 6 — 3 3 x — 17 — 3 3 ‘ = = e x + 1 ‘ + x 3 3 ‘ — 6 — 3 3 x ‘ — 17 — 3 3 ‘ = e x + 1 + x 2 — 6 — 3 3 y ‘ ( x 0 ) = y ‘ ( — 1 ) = e — 1 + 1 + — 1 2 — 6 — 3 3 = 3 3

Значение f ’ ( x ) в точке касания является угловым коэффициентом касательной, который равняется тангенсу наклона.

Тогда k x = t g α x = y ‘ ( x 0 ) = 3 3

Отсюда следует, что α x = a r c t g 3 3 = π 6

Ответ: уравнение касательной приобретает вид

y = f ‘ ( x 0 ) · x — x 0 + f ( x 0 ) y = 3 3 ( x + 1 ) — 3 y = 3 3 x — 9 — 3 3

Для наглядности приведем пример в графической иллюстрации.

Черный цвет используется для графика исходной функции, синий цвет – изображение касательной, красная точка – точка касания. Рисунок, располагаемый справа, показывает в увеличенном виде.

Выяснить наличие существования касательной к графику заданной функции
y = 3 · x — 1 5 + 1 в точке с координатами ( 1 ; 1 ) . Составить уравнение и определить угол наклона.

Решение

По условию имеем, что областью определения заданной функции считается множество всех действительных чисел.

Перейдем к нахождению производной

y ‘ = 3 · x — 1 5 + 1 ‘ = 3 · 1 5 · ( x — 1 ) 1 5 — 1 = 3 5 · 1 ( x — 1 ) 4 5

Если x 0 = 1 , тогда f ’ ( x ) не определена, но пределы записываются как lim x → 1 + 0 3 5 · 1 ( x — 1 ) 4 5 = 3 5 · 1 ( + 0 ) 4 5 = 3 5 · 1 + 0 = + ∞ и lim x → 1 — 0 3 5 · 1 ( x — 1 ) 4 5 = 3 5 · 1 ( — 0 ) 4 5 = 3 5 · 1 + 0 = + ∞ , что означает существование вертикальной касательной в точке ( 1 ; 1 ) .

Ответ: уравнение примет вид х = 1 , где угол наклона будет равен π 2 .

Для наглядности изобразим графически.

Найти точки графика функции y = 1 15 x + 2 3 — 4 5 x 2 — 16 5 x — 26 5 + 3 x + 2 , где

  1. Касательная не существует;
  2. Касательная располагается параллельно о х ;
  3. Касательная параллельна прямой y = 8 5 x + 4 .

Решение

Необходимо обратить внимание на область определения. По условию имеем, что функция определена на множестве всех действительных чисел. Раскрываем модуль и решаем систему с промежутками x ∈ — ∞ ; 2 и [ — 2 ; + ∞ ) . Получаем, что

y = — 1 15 x 3 + 18 x 2 + 105 x + 176 , x ∈ — ∞ ; — 2 1 15 x 3 — 6 x 2 + 9 x + 12 , x ∈ [ — 2 ; + ∞ )

Необходимо продифференцировать функцию. Имеем, что

y ‘ = — 1 15 x 3 + 18 x 2 + 105 x + 176 ‘ , x ∈ — ∞ ; — 2 1 15 x 3 — 6 x 2 + 9 x + 12 ‘ , x ∈ [ — 2 ; + ∞ ) ⇔ y ‘ = — 1 5 ( x 2 + 12 x + 35 ) , x ∈ — ∞ ; — 2 1 5 x 2 — 4 x + 3 , x ∈ [ — 2 ; + ∞ )

Когда х = — 2 , тогда производная не существует, потому что односторонние пределы не равны в этой точке:

lim x → — 2 — 0 y ‘ ( x ) = lim x → — 2 — 0 — 1 5 ( x 2 + 12 x + 35 = — 1 5 ( — 2 ) 2 + 12 ( — 2 ) + 35 = — 3 lim x → — 2 + 0 y ‘ ( x ) = lim x → — 2 + 0 1 5 ( x 2 — 4 x + 3 ) = 1 5 — 2 2 — 4 — 2 + 3 = 3

Вычисляем значение функции в точке х = — 2 , где получаем, что

  1. y ( — 2 ) = 1 15 — 2 + 2 3 — 4 5 ( — 2 ) 2 — 16 5 ( — 2 ) — 26 5 + 3 — 2 + 2 = — 2 , то есть касательная в точке ( — 2 ; — 2 ) не будет существовать.
  2. Касательная параллельна о х , когда угловой коэффициент равняется нулю. Тогда k x = t g α x = f ‘ ( x 0 ) . То есть необходимо найти значения таких х , когда производная функции обращает ее в ноль. То есть значения f ’ ( x ) и будут являться точками касания, где касательная является параллельной о х .

Когда x ∈ — ∞ ; — 2 , тогда — 1 5 ( x 2 + 12 x + 35 ) = 0 , а при x ∈ ( — 2 ; + ∞ ) получаем 1 5 ( x 2 — 4 x + 3 ) = 0 .

— 1 5 ( x 2 + 12 x + 35 ) = 0 D = 12 2 — 4 · 35 = 144 — 140 = 4 x 1 = — 12 + 4 2 = — 5 ∈ — ∞ ; — 2 x 2 = — 12 — 4 2 = — 7 ∈ — ∞ ; — 2 1 5 ( x 2 — 4 x + 3 ) = 0 D = 4 2 — 4 · 3 = 4 x 3 = 4 — 4 2 = 1 ∈ — 2 ; + ∞ x 4 = 4 + 4 2 = 3 ∈ — 2 ; + ∞

Вычисляем соответствующие значения функции

y 1 = y — 5 = 1 15 — 5 + 2 3 — 4 5 — 5 2 — 16 5 — 5 — 26 5 + 3 — 5 + 2 = 8 5 y 2 = y ( — 7 ) = 1 15 — 7 + 2 3 — 4 5 ( — 7 ) 2 — 16 5 — 7 — 26 5 + 3 — 7 + 2 = 4 3 y 3 = y ( 1 ) = 1 15 1 + 2 3 — 4 5 · 1 2 — 16 5 · 1 — 26 5 + 3 1 + 2 = 8 5 y 4 = y ( 3 ) = 1 15 3 + 2 3 — 4 5 · 3 2 — 16 5 · 3 — 26 5 + 3 3 + 2 = 4 3

Отсюда — 5 ; 8 5 , — 4 ; 4 3 , 1 ; 8 5 , 3 ; 4 3 считаются искомыми точками графика функции.

Рассмотрим графическое изображение решения.

Черная линия – график функции, красные точки – точки касания.

  1. Когда прямые располагаются параллельно, то угловые коэффициенты равны. Тогда необходимо заняться поиском точек графика функции, где угловой коэффициент будет равняться значению 8 5 . Для этого нужно решить уравнение вида y ‘ ( x ) = 8 5 . Тогда, если x ∈ — ∞ ; — 2 , получаем, что — 1 5 ( x 2 + 12 x + 35 ) = 8 5 , а если x ∈ ( — 2 ; + ∞ ) , тогда 1 5 ( x 2 — 4 x + 3 ) = 8 5 .

Первое уравнение не имеет корней, так как дискриминант меньше нуля. Запишем, что

— 1 5 x 2 + 12 x + 35 = 8 5 x 2 + 12 x + 43 = 0 D = 12 2 — 4 · 43 = — 28 0

Другое уравнение имеет два действительных корня, тогда

1 5 ( x 2 — 4 x + 3 ) = 8 5 x 2 — 4 x — 5 = 0 D = 4 2 — 4 · ( — 5 ) = 36 x 1 = 4 — 36 2 = — 1 ∈ — 2 ; + ∞ x 2 = 4 + 36 2 = 5 ∈ — 2 ; + ∞

Перейдем к нахождению значений функции. Получаем, что

y 1 = y ( — 1 ) = 1 15 — 1 + 2 3 — 4 5 ( — 1 ) 2 — 16 5 ( — 1 ) — 26 5 + 3 — 1 + 2 = 4 15 y 2 = y ( 5 ) = 1 15 5 + 2 3 — 4 5 · 5 2 — 16 5 · 5 — 26 5 + 3 5 + 2 = 8 3

Точки со значениями — 1 ; 4 15 , 5 ; 8 3 являются точками, в которых касательные параллельны прямой y = 8 5 x + 4 .

Ответ: черная линия – график функции, красная линия – график y = 8 5 x + 4 , синяя линия – касательные в точках — 1 ; 4 15 , 5 ; 8 3 .

Возможно существование бесконечного количества касательных для заданных функций.

Написать уравнения всех имеющихся касательных функции y = 3 cos 3 2 x — π 4 — 1 3 , которые располагаются перпендикулярно прямой y = — 2 x + 1 2 .

Решение

Для составления уравнения касательной необходимо найти коэффициент и координаты точки касания, исходя из условия перпендикулярности прямых. Определение звучит так: произведение угловых коэффициентов, которые перпендикулярны прямым, равняется — 1 , то есть записывается как k x · k ⊥ = — 1 . Из условия имеем, что угловой коэффициент располагается перпендикулярно прямой и равняется k ⊥ = — 2 , тогда k x = — 1 k ⊥ = — 1 — 2 = 1 2 .

Теперь необходимо найти координаты точек касания. Нужно найти х , после чего его значение для заданной функции. Отметим, что из геометрического смысла производной в точке
x 0 получаем, что k x = y ‘ ( x 0 ) . Из данного равенства найдем значения х для точек касания.

y ‘ ( x 0 ) = 3 cos 3 2 x 0 — π 4 — 1 3 ‘ = 3 · — sin 3 2 x 0 — π 4 · 3 2 x 0 — π 4 ‘ = = — 3 · sin 3 2 x 0 — π 4 · 3 2 = — 9 2 · sin 3 2 x 0 — π 4 ⇒ k x = y ‘ ( x 0 ) ⇔ — 9 2 · sin 3 2 x 0 — π 4 = 1 2 ⇒ sin 3 2 x 0 — π 4 = — 1 9

Это тригонометрическое уравнение будет использовано для вычисления ординат точек касания.

3 2 x 0 — π 4 = a r c sin — 1 9 + 2 πk или 3 2 x 0 — π 4 = π — a r c sin — 1 9 + 2 πk

3 2 x 0 — π 4 = — a r c sin 1 9 + 2 πk или 3 2 x 0 — π 4 = π + a r c sin 1 9 + 2 πk

x 0 = 2 3 π 4 — a r c sin 1 9 + 2 πk или x 0 = 2 3 5 π 4 + a r c sin 1 9 + 2 πk , k ∈ Z

Z — множество целых чисел.

Найдены х точек касания. Теперь необходимо перейти к поиску значений у :

y 0 = 3 cos 3 2 x 0 — π 4 — 1 3

y 0 = 3 · 1 — sin 2 3 2 x 0 — π 4 — 1 3 или y 0 = 3 · — 1 — sin 2 3 2 x 0 — π 4 — 1 3

y 0 = 3 · 1 — — 1 9 2 — 1 3 или y 0 = 3 · — 1 — — 1 9 2 — 1 3

y 0 = 4 5 — 1 3 или y 0 = — 4 5 + 1 3

Отсюда получаем, что 2 3 π 4 — a r c sin 1 9 + 2 πk ; 4 5 — 1 3 , 2 3 5 π 4 + a r c sin 1 9 + 2 πk ; — 4 5 + 1 3 являются точками касания.

Ответ: необходимы уравнения запишутся как

y = 1 2 x — 2 3 π 4 — a r c sin 1 9 + 2 πk + 4 5 — 1 3 , y = 1 2 x — 2 3 5 π 4 + a r c sin 1 9 + 2 πk — 4 5 + 1 3 , k ∈ Z

Для наглядного изображения рассмотрим функцию и касательную на координатной прямой.

Рисунок показывает, что расположение функции идет на промежутке [ — 10 ; 10 ] , где черная прямя – график функции, синие линии – касательные, которые располагаются перпендикулярно заданной прямой вида y = — 2 x + 1 2 . Красные точки – это точки касания.

Касательная к окружности, эллипсу, гиперболе, параболе

Канонические уравнения кривых 2 порядка не являются однозначными функциями. Уравнения касательных для них составляются по известным схемам.

Касательная к окружности

Для задания окружности с центром в точке x c e n t e r ; y c e n t e r и радиусом R применяется формула x — x c e n t e r 2 + y — y c e n t e r 2 = R 2 .

Данное равенство может быть записано как объединение двух функций:

y = R 2 — x — x c e n t e r 2 + y c e n t e r y = — R 2 — x — x c e n t e r 2 + y c e n t e r

Первая функция располагается вверху, а вторая внизу, как показано на рисунке.

Для составления уравнения окружности в точке x 0 ; y 0 , которая располагается в верхней или нижней полуокружности, следует найти уравнение графика функции вида y = R 2 — x — x c e n t e r 2 + y c e n t e r или y = — R 2 — x — x c e n t e r 2 + y c e n t e r в указанной точке.

Когда в точках x c e n t e r ; y c e n t e r + R и x c e n t e r ; y c e n t e r — R касательные могут быть заданы уравнениями y = y c e n t e r + R и y = y c e n t e r — R , а в точках x c e n t e r + R ; y c e n t e r и
x c e n t e r — R ; y c e n t e r будут являться параллельными о у , тогда получим уравнения вида x = x c e n t e r + R и x = x c e n t e r — R .

Касательная к эллипсу

Когда эллипс имеет центр в точке x c e n t e r ; y c e n t e r с полуосями a и b , тогда он может быть задан при помощи уравнения x — x c e n t e r 2 a 2 + y — y c e n t e r 2 b 2 = 1 .

Эллипс и окружность могут быть обозначаться при помощи объединения двух функций, а именно: верхнего и нижнего полуэллипса. Тогда получаем, что

y = b a · a 2 — ( x — x c e n t e r ) 2 + y c e n t e r y = — b a · a 2 — ( x — x c e n t e r ) 2 + y c e n t e r

Если касательные располагаются на вершинах эллипса, тогда они параллельны о х или о у . Ниже для наглядности рассмотрим рисунок.

Написать уравнение касательной к эллипсу x — 3 2 4 + y — 5 2 25 = 1 в точках со значениями x равного х = 2 .

Решение

Необходимо найти точки касания, которые соответствуют значению х = 2 . Производим подстановку в имеющееся уравнение эллипса и получаем, что

x — 3 2 4 x = 2 + y — 5 2 25 = 1 1 4 + y — 5 2 25 = 1 ⇒ y — 5 2 = 3 4 · 25 ⇒ y = ± 5 3 2 + 5

Тогда 2 ; 5 3 2 + 5 и 2 ; — 5 3 2 + 5 являются точками касания, которые принадлежат верхнему и нижнему полуэллипсу.

Перейдем к нахождению и разрешению уравнения эллипса относительно y . Получим, что

x — 3 2 4 + y — 5 2 25 = 1 y — 5 2 25 = 1 — x — 3 2 4 ( y — 5 ) 2 = 25 · 1 — x — 3 2 4 y — 5 = ± 5 · 1 — x — 3 2 4 y = 5 ± 5 2 4 — x — 3 2

Очевидно, что верхний полуэллипс задается с помощью функции вида y = 5 + 5 2 4 — x — 3 2 , а нижний y = 5 — 5 2 4 — x — 3 2 .

Применим стандартный алгоритм для того, чтобы составить уравнение касательной к графику функции в точке. Запишем, что уравнение для первой касательной в точке 2 ; 5 3 2 + 5 будет иметь вид

y ‘ = 5 + 5 2 4 — x — 3 2 ‘ = 5 2 · 1 2 4 — ( x — 3 ) 2 · 4 — ( x — 3 ) 2 ‘ = = — 5 2 · x — 3 4 — ( x — 3 ) 2 ⇒ y ‘ ( x 0 ) = y ‘ ( 2 ) = — 5 2 · 2 — 3 4 — ( 2 — 3 ) 2 = 5 2 3 ⇒ y = y ‘ ( x 0 ) · x — x 0 + y 0 ⇔ y = 5 2 3 ( x — 2 ) + 5 3 2 + 5

Получаем, что уравнение второй касательной со значением в точке
2 ; — 5 3 2 + 5 принимает вид

y ‘ = 5 — 5 2 4 — ( x — 3 ) 2 ‘ = — 5 2 · 1 2 4 — ( x — 3 ) 2 · 4 — ( x — 3 ) 2 ‘ = = 5 2 · x — 3 4 — ( x — 3 ) 2 ⇒ y ‘ ( x 0 ) = y ‘ ( 2 ) = 5 2 · 2 — 3 4 — ( 2 — 3 ) 2 = — 5 2 3 ⇒ y = y ‘ ( x 0 ) · x — x 0 + y 0 ⇔ y = — 5 2 3 ( x — 2 ) — 5 3 2 + 5

Графически касательные обозначаются так:

Касательная к гиперболе

Когда гипербола имеет центр в точке x c e n t e r ; y c e n t e r и вершины x c e n t e r + α ; y c e n t e r и x c e n t e r — α ; y c e n t e r , имеет место задание неравенства x — x c e n t e r 2 α 2 — y — y c e n t e r 2 b 2 = 1 , если с вершинами x c e n t e r ; y c e n t e r + b и x c e n t e r ; y c e n t e r — b , тогда задается при помощи неравенства x — x c e n t e r 2 α 2 — y — y c e n t e r 2 b 2 = — 1 .

Гипербола может быть представлена в виде двух объединенных функций вида

y = b a · ( x — x c e n t e r ) 2 — a 2 + y c e n t e r y = — b a · ( x — x c e n t e r ) 2 — a 2 + y c e n t e r или y = b a · ( x — x c e n t e r ) 2 + a 2 + y c e n t e r y = — b a · ( x — x c e n t e r ) 2 + a 2 + y c e n t e r

В первом случае имеем, что касательные параллельны о у , а во втором параллельны о х .

Отсюда следует, что для того, чтобы найти уравнение касательной к гиперболе, необходимо выяснить, какой функции принадлежит точка касания. Чтобы определить это, необходимо произвести подстановку в уравнения и проверить их на тождественность.

Составить уравнение касательной к гиперболе x — 3 2 4 — y + 3 2 9 = 1 в точке 7 ; — 3 3 — 3 .

Решение

Необходимо преобразовать запись решения нахождения гиперболы при помощи 2 функций. Получим, что

x — 3 2 4 — y + 3 2 9 = 1 ⇒ y + 3 2 9 = x — 3 2 4 — 1 ⇒ y + 3 2 = 9 · x — 3 2 4 — 1 ⇒ y + 3 = 3 2 · x — 3 2 — 4 и л и y + 3 = — 3 2 · x — 3 2 — 4 ⇒ y = 3 2 · x — 3 2 — 4 — 3 y = — 3 2 · x — 3 2 — 4 — 3

Необходимо выявить, к какой функции принадлежит заданная точка с координатами 7 ; — 3 3 — 3 .

Очевидно, что для проверки первой функции необходимо y ( 7 ) = 3 2 · ( 7 — 3 ) 2 — 4 — 3 = 3 3 — 3 ≠ — 3 3 — 3 , тогда точка графику не принадлежит, так как равенство не выполняется.

Для второй функции имеем, что y ( 7 ) = — 3 2 · ( 7 — 3 ) 2 — 4 — 3 = — 3 3 — 3 ≠ — 3 3 — 3 , значит, точка принадлежит заданному графику. Отсюда следует найти угловой коэффициент.

y ‘ = — 3 2 · ( x — 3 ) 2 — 4 — 3 ‘ = — 3 2 · x — 3 ( x — 3 ) 2 — 4 ⇒ k x = y ‘ ( x 0 ) = — 3 2 · x 0 — 3 x 0 — 3 2 — 4 x 0 = 7 = — 3 2 · 7 — 3 7 — 3 2 — 4 = — 3

Ответ: уравнение касательной можно представить как

y = — 3 · x — 7 — 3 3 — 3 = — 3 · x + 4 3 — 3

Наглядно изображается так:

Касательная к параболе

Чтобы составить уравнение касательной к параболе y = a x 2 + b x + c в точке x 0 , y ( x 0 ) , необходимо использовать стандартный алгоритм, тогда уравнение примет вид y = y ‘ ( x 0 ) · x — x 0 + y ( x 0 ) . Такая касательная в вершине параллельна о х .

Следует задать параболу x = a y 2 + b y + c как объединение двух функций. Поэтому нужно разрешить уравнение относительно у . Получаем, что

x = a y 2 + b y + c ⇔ a y 2 + b y + c — x = 0 D = b 2 — 4 a ( c — x ) y = — b + b 2 — 4 a ( c — x ) 2 a y = — b — b 2 — 4 a ( c — x ) 2 a

Графически изобразим как:

Для выяснения принадлежности точки x 0 , y ( x 0 ) функции, нежно действовать по стандартному алгоритму. Такая касательная будет параллельна о у относительно параболы.

Написать уравнение касательной к графику x — 2 y 2 — 5 y + 3 , когда имеем угол наклона касательной 150 ° .

Решение

Начинаем решение с представления параболы в качестве двух функций. Получим, что

— 2 y 2 — 5 y + 3 — x = 0 D = ( — 5 ) 2 — 4 · ( — 2 ) · ( 3 — x ) = 49 — 8 x y = 5 + 49 — 8 x — 4 y = 5 — 49 — 8 x — 4

Значение углового коэффициента равняется значению производной в точке x 0 этой функции и равняется тангенсу угла наклона.

k x = y ‘ ( x 0 ) = t g α x = t g 150 ° = — 1 3

Отсюда определим значение х для точек касания.

Первая функция запишется как

y ‘ = 5 + 49 — 8 x — 4 ‘ = 1 49 — 8 x ⇒ y ‘ ( x 0 ) = 1 49 — 8 x 0 = — 1 3 ⇔ 49 — 8 x 0 = — 3

Очевидно, что действительных корней нет, так как получили отрицательное значение. Делаем вывод, что касательной с углом 150 ° для такой функции не существует.

Вторая функция запишется как

y ‘ = 5 — 49 — 8 x — 4 ‘ = — 1 49 — 8 x ⇒ y ‘ ( x 0 ) = — 1 49 — 8 x 0 = — 1 3 ⇔ 49 — 8 x 0 = — 3 x 0 = 23 4 ⇒ y ( x 0 ) = 5 — 49 — 8 · 23 4 — 4 = — 5 + 3 4

Имеем, что точки касания — 23 4 ; — 5 + 3 4 .

Ответ: уравнение касательной принимает вид

Касательная и нормаль к графику функции

Основные формулы

Пусть на некотором интервале X задана функция . Нас интересуют геометрические характеристики графика этой функции в некоторой заданной точке при значении аргумента , где . Пусть функция имеет в производную, которую будем обозначать как . Тогда через точку мы можем провести касательную к графику. Тангенс угла α между осью абсцисс x и касательной равен производной функции в точке :
(1) .
А само уравнение касательной имеет вид:
(2) .
В аналитической геометрии тангенс угла между прямой и осью абсцисс называют угловым коэффициентом прямой. Таким образом производная равна угловому коэффициенту касательной в .
См. Геометрический смысл производной

Прямая, перпендикулярная касательной, проведенной через точку , называется нормалью к графику функции в этой точке. Уравнение нормали имеет вид:
(3) .
См. Уравнение прямой с угловым коэффициентом ⇓

Пусть две кривые и пересекаются в точке . Тогда угол φ между касательными к этим кривым в точке называется углом между кривыми. Он определяется по формуле:
(4) , где .
Отсюда .
при .
Вывод формулы ⇓

Определения

Здесь мы приводим определения, которые встречаются в литературе, и имеют отношение к касательной и нормали. Вывод формул приводится в примере 1 ⇓.

Определение касательной приводится здесь. Уравнение касательной:
.

Касательная TM0, нормаль M0N, подкасательная TP, поднормаль PN. Нормалью к графику функции в точке называется прямая, перпендикулярная касательной, проведенной через эту точку. Уравнение нормали:
.
Отрезком касательной называют отрезок между точкой пересечения касательной с осью абсцисс и точкой .
.
Отрезком нормали называют отрезок между точкой пересечения нормали с осью абсцисс и точкой .
.
Подкасательной называют отрезок между точкой пересечения касательной с осью абсцисс и проекции точки на эту ось.
.
Поднормалью называют отрезок между точкой пересечения нормали с осью абсцисс и проекции точки на эту ось.
.
Углом между кривыми в точке их пересечения называют угол между касательными к кривым, проведенных через точку .

Полезные формулы из аналитической геометрии

Далее приводятся некоторые сведения из аналитической геометрии, которые могут оказаться полезными при решении задач.

Уравнение прямой, проходящей через две заданные точки и :
.
Здесь – направляющий вектор прямой.

Умножив это уравнение на , получим уравнение прямой в другом виде:
.
Здесь – вектор нормали прямой. Тогда само уравнение означает равенство нулю скалярного произведения векторов и .

Уравнение прямой, проходящей через точку параллельно вектору имеет вид:
.
Вектор называется направляющим вектором данной прямой. Это уравнение можно написать в параметрическом виде, введя параметр t :

Уравнение прямой, проходящей через точку перпендикулярно вектору имеет вид:
.
Вектор называется вектором нормали данной прямой.

Уравнение прямой с угловым коэффициентом k , проходящей через точку :
.
Угол α между прямой и осью x определяется по формуле:
.
Если две прямые взаимно перпендикулярны, то их угловые коэффициенты и связаны соотношением:
.

Уравнение прямой в отрезках, пересекающей оси координат в точках :
.

Примеры решения задач

Все примеры Ниже рассмотрены примеры решений следующих задач.
1. Найти уравнения касательной и нормали к кривой в точке . Найти длины отрезков касательной, нормали, подкасательной и поднормали. Решение ⇓
2. Составить уравнения касательной и нормали к циссоиде, заданной в параметрическом виде
, проведенных в точке . Решение ⇓
3. Заданной в неявном виде . Решение ⇓
4. Найти угол между кривыми и Решение ⇓

Пример 1

Составить уравнения касательной и нормали к кривой в точке . Найти длины отрезков касательной, нормали, подкасательной и поднормали.

Находим значение функции при :
.

Находим производную:
.
Находим производную в точке :
;
.

Находим уравнение касательной по формуле (2):
;
;
;
– уравнение касательной.
Строим касательную на графике. Поскольку касательная – это прямая, то нам нужно знать положения двух ее точек, и провести через них прямую.
При ;
при .
Проводим касательную через точки и .

Касательная и нормаль к графику функции y=x 2 в точке M0(1;1).

Найдем угол α между касательной и осью абсцисс по формуле (1):
.
Подставляем :
;
.

Находим уравнение нормали по формуле (3):
;
;
;
;
;
– уравнение нормали.
Строим нормаль по двум точкам.
При ;
при .
Проводим нормаль через точки и .

Находим длину отрезка касательной . Из прямоугольника имеем:
.
Поясним использованную формулу. Поскольку , то . Тогда
.
Подставляем :
.

Находим длину отрезка подкасательной . Из прямоугольника имеем:
.
Подставляем :
.

Находим длину отрезка нормали . Поскольку и , то треугольники и подобны. Тогда . Из прямоугольника имеем:
.
Подставляем :
.

Находим длину отрезка поднормали . Из прямоугольника имеем:
.

Примечание.
При выводе формул, можно сначала определить длины отрезков подкасательной и поднормали, а затем из прямоугольников, по теореме Пифагора, найти длины отрезков касательной и нормали:
;
.

Уравнение касательной: ; уравнение нормали: ;
длина отрезка касательной: ; длина отрезка нормали: ; длина подкасательной: ; длина поднормали: .

Пример 2

Составить уравнения касательной и нормали к циссоиде, заданной в параметрическом виде , проведенных в точке .

Находим значения переменных при .
;
.
Обозначим эту точку как .

Находим производные переменных x и y по параметру t .
;
;
;
;
.

Подставляя , находим производную y по x в точке .
.

Касательная и нормаль к циссоиде в точке (2;2).

Применяя формулу (2), находим уравнение касательной к циссоиде, проходящей через точку .
;
;
;
.

Применяя формулу (3), находим уравнение нормали к циссоиде в точке .
;
;
;
.

Уравнение касательной: .
Уравнение нормали: .

Пример 3

Составить уравнения касательной и нормали к циссоиде, заданной в неявном виде:
(П3) ,
проведенных в точке .

Для получения уравнение касательной и нормали, нам нужно знать значение производной функции в заданной точке. Функция (П3) задана неявно. Поэтому применяем правило дифференцирования неявной функции. Для этого дифференцируем (П3) по x , считая, что y является функцией от x .
;
;
;
.
Отсюда
.

Находим производную в заданной точке, подставляя .
;
.

Находим уравнение касательной по формуле (2).
;
;
;
.

Находим уравнение нормали по формуле (3).
;
;
;
.

Касательная и нормаль к циссоиде изображены на рисунке ⇑.

Уравнение касательной: .
Уравнение нормали: .

Пример 4

Найти угол между кривыми и .

Найдем множество точек пересечения кривых, решая систему уравнений.

Левые части равны. Приравниваем правые части и выполняем преобразования.
;
(П4) .
Поскольку функция строго монотонна, то уравнение (П4) имеет один корень:
.
При . Кривые пересекаются в единственной точке . Обозначим ее как , где .

Введем обозначения для функций, с помощью которых заданы кривые:
.
Найдем их производные.
;
.
Найдем значения производных в точке , подставляя .
;
.

Ниже приводятся графики функций ⇓ и вывод формулы угла между кривыми.

Вывод формулы для угла между кривыми

Изложим вывод формулы (4). Для иллюстрации используем только что рассмотренный пример ⇑, в котором .

Рассмотрим две кривые, заданные уравнениями и , и пересекающиеся в некоторой точке . Докажем, что угол между кривыми определяется по формуле (4):
, где .
Или ;
при .

Проведем касательные к графикам функций в точке . Углы, которые образуют касательные с осью x обозначим как и . За положительное направление выберем направление против часовой стрелки. На рисунке . Считаем, что значения углов принадлежат интервалам . Согласно геометрическому смыслу производной,
.

В аналитической геометрии принято, что угол φ между прямыми равен наименьшему значению угла между ними.
Если , то ;
если , то .
Таким образом величина угла φ между касательными может находиться только в пределах
(Ф2) .

На рисунке угол между лучами и больше 90°, а между лучами и – меньше. Поэтому .

При доказательстве мы будем использовать соотношение:
, которое выполняется при .
Тогда в силу (Ф2),
.
Случай мы рассмотрим отдельно.

1) Пусть .
Тогда угол между прямыми . И мы имеем:
.
В конце мы подставили (Ф1).

2) Пусть .
Тогда ; . Поэтому . Это можно записать так: . Также применим формулу: . В результате получаем:

.

Этот случай изображен на рисунке ⇑.

3) Пусть .
При этом касательные взаимно перпендикулярны, . В этом случае , что указано в (4).

Использованная литература:
П.Е. Данько, А.Г. Попов, Т.Я.Кожевникова. Высшая математика в упражнениях и задачах. Часть 1. Москва, Высшая школа, 1980.
Л.Д. Кудрявцев, А.Д. Кутасов, В.И. Чехлов, М.И. Шабунин. Сборник задач по математическому анализу. Том 1. Москва, Физматлит, 2003.

Автор: Олег Одинцов . Опубликовано: 30-06-2021

источники:

http://zaochnik.com/spravochnik/matematika/proizvodnye/kasatelnaja-k-grafiku-funktsii-v-tochke/

http://1cov-edu.ru/mat-analiz/proizvodnaya/kasatelnaya-i-normal-k-grafiku-funktsii/

Понравилась статья? Поделить с друзьями:
  • Как найти уличных музыкантов
  • Как составить иерархию системы
  • Как найти свой домашний адрес
  • Как найти галерею в вайбере
  • Как составить виш лист подарков